You are on page 1of 249
Wl Mechanics & Hydraulics 1 Edit DIEGO INOCENCIO T. GIL Civil Engineer BSCE, LIT ~ Magna Cum Laude ‘rd Place, CE Board November 1989 ny Director & Re y Center ea Ml jeneral Engineering Sciences nila Fluid Mechanics and Hydraulics tion Copyright © 1997, 1999, 2003 by Diego Inocencio Tapang Gillesanta No part of this book may be twutbout the prior permission of the amor ISBN 971-8614-28-1 Printed by, GPP Gillesania Printing Press Grmoe City, Leyte Philippines Cover design by the au prieiace as The cardinal objective of this book is to provide reference to Engineering students taking-up Fluid Mechanics and Hydraulics. This may also serve as a guide to engineering students who will be taking, the licensure examination given by the PRC. The book has 9 chapters. Each chapter presents the principles and formulas involved, followed by solved problems and supplementary problems. Each step in the solution is carefully explained to ensure that it will be readily understood. Some problems are even solved in several methods to give the reader a choice on the type of solution he may adopt. To provide the reader easy access to the different topics, the book includes index. Most of the materials in this book have been used in my review classes. The choice of these materials was guided by their effectiveness as tested in my classes Lwish to thank all my friends and relatives who inspired me in writing my books and especially to my children and beloved wife Imelda who is very supportive to me. I will appreciate any errors pointed out and will welcome any suggestion for further improvement. DIEGO INOCENCIO T. GILLESANIA Cebu City, Philippines To my mother Iuminada, my wife Imelda, and our Children Kim Deunice, Ken Dainiel, and Karla Denise Preface Dedic: TABLE OF CONTENTS CHAPTER 1 Properties of Fluid Types of Fluid Mass Density. Specific Volume. Unit Weight or Specific Weight Specific Gravity Viscosity... Kinematic Viscosity seein Surface Tension... Capillarity Compressibility Pressure Disturbances... Boney Vapor Pressure =a SOLYED PROBLEMS. ee SUPPLEMENTARY PROBLEMS anges in Ideal Gas CHAPTER 2 Principles of Hydrostatics Unit Pressure Pascal's Law. Absolute and Gage Pressures. Variations in Pressure Pressure below Layers of Different Liquids Pressure Head. Manometers SOLVED PROBLEMS. SUPPLEMENTARY PROBLEMS ——— vii viii 9 0) 11 to 23, 24 to 26 ag 7 7 29 95 to 68 69 to 72, ii TABLE OF CONTENTS TABLE OF CONTENTS iii CHAPTER 3 CHAPTER 5 Total Hydrostatic Force on Surfaces 3 Fundamentals of Fluid Flow SAT Total Hydrostatic Force on Plane Surface 73 Discharge. ao Properties of Common Geometric Shapes. % Definition of Terms. 241 Total Hydrostatic Force on Curved Surface. 78 Energy and Head..... 244 Power and Efficiency 245 Dams 81 Bernoulli’s Energy Theorem. - 246 Types of Dams 81 Energy and Hydraulic Grade Lines 248 Analysis of Gravity Dams. 84 SOLVED PROBLEMS 250 to 278 Buoyancy 88 SUPPLEMENTARY PROBLEMS 274 to 276 Archimedes’ Principles 88 Statical Stability of Floating Bodies 90 ‘Stress on Thin-Walled Pressure Vessels 96 Cylindrical Tank. 96 CHAPTER 6 Spherical Shell 98 Fluid Flow Measurement Wood Stave Pipes... 98 Device Coefficients ‘ SOLVED PROBLEMS. 99 to 195 Head lost in Measuring Devices SUPPLEMENTARY PROBLEMS 196 to 200 Orifice i Values of H for Various Conditions Contraction of the Jet. Orifice under Low Heads. CHAPTER 4 Venturi Meter Relative Equilibrium of Liquids 201 Nozle su Rectilinear Translation 201 Pitot Tube Horizontal Motion 201 Gates Inclined Motion 202 Tubes. Vertical Motion 203 Unsteady Flow (Orifice) Rotation 203 Weir: Senn Volume of Paraboloid. 205 Classification of Weirs. Liquid Surface Conditions 206 Rectangular Weif...nm0 SOLVED PROBLEMS 210 to 240 Contracted Rectangular Weit Triangular Weirs Trapezoidal Weir wn. Cipolletti Weir Suttro Weir: Submerged Weir Unsteady Flow SOLVED PROBLEMS SUPPLEMENTARY PROBLEMS CHAPTER 7 Fluid Flow in Pipes. Definitions Reynolds Number. . Velocity Distribution in Pipes Shearing Stress in Pipes... Head Losses in Pipe Flow. Major Head L Darcy-Weisbach Formula. Value of f. Moody Diagram Manning Formula Hazen Williams Formula Minor Head Loss. Sudden Enlargement Gradual Enlargement Sudden Contraction. Bends and Standard Fittings. Pipe Discharging from Reservoir Pipe Connecting Two Reservoirs. Pipes in Series and Parallel Equivalent Pipe Reservoir Problems. Pipe Networks. i SOLVED PROBLEMS.... TABLE OF CONTENTS SUPPLEMENTARY PROBLEMS TABLE OF CONTENTS CHAPTER 8 Open Channel Specific Energy .. Chézy Formula... ‘Kutter and Gunguillet Formula ‘Manning Formula Bazin Formula, at Powell Equation... Uniform Flow Boundary Shear Stress Normal Depth... Most Efficient Sections Proportions for Most Efficient Sections Rectangular Section... Trapezoidal Section. Triangular Section Circular Sections... Velocity Distribution in Open Channel Alternate Stages of Flow Froude Number... Critfeal Depth. Non-Uniform or Varied Flow Hydraulic Jump. Flow around Channel Bends... SOLVED PROBLEMS. SUPPLEMENTARY PROBLEMS v EE ve 481 481 482 483 483 483 434 485 485 486 486 437 en ABT sn 487 489 490 491 491 492 492 495 497 500 “501 to 547 547 to 550 vi TABLE OF CONTENTS Se en CHAPTER 9 Hydrodynamics. ee Force against Fixed Flat Plates Force against Fixed Curved Vanes Force against Moving Vanes Work Done on Moving Vanes Force Developed on Closed Conduit Drag and Lift. Terminal Velocity. Water Hammer... SOLVED PROBLEMS SUPPLEMENTARY PROBLEMS APPENDIX 557 559 560 563 to 597 597 to 598. Properties of Fluids and Conversion Factors 599 Table A - 1: Viscosity and Density of Water at 1 atm 599 Table A - 2: Viscosity and Density of Air at 1 atmreennu- 600 Table A - 3: Properties of Common Liquids at 1 atm & 20°C...601 Table A - 4: Froperties of Common Gases at 1 atm & 20°C..... 601 Table A - 5: Surface Tension, Vapor Pressure, and Sound Speed of Water 602 Table A - 6; Properties of Standard Atmosphere 608 Table A - 7: Conversion Factors from BG to SI Units 604 Table A - 8: Other Conversion Factors 605 INDEX 1-1IV FLUID MECHANICS & HYDRAULICS Chapter 1 Properties of Fluids (CHAPTER ONE 1 Properties of Fluids FLUID MECHANICS & HYDRAULICS Fluid Mechanics is a physical science dealing with the action of fluids at rest or in motion, and with applications and devices in engineering using fluids: Fluid mechanics can be subdivided into two major areas, fluid statics, which deals with fluids at rest, andl fluid dynamics, concerned with fluids in motion “The term hydrodynamics is applied to the flow of liquids or to low-velocity gas flows where the gas can be considered as bei z essentially incompressible, yds deals with the application of fuld mechanics to engineeting cetng guide urually ater or of Hydraulics deals with such problems ahs few of lida trough pipes orn open channels, the design a storage tse of guid, such as ozzes, valve, el, and wines ‘TYPES OF FLUID Fluids are generally divided into two categories: ideal fluids and real fluids, Ideal fluids ‘Assumed to have no viscosity (and hence, no resistance to shear) «+ Incompressible + Have uniforni velocity when flowin No friction between moving layers of fluid + No eddy currents or turbulence Real fluids «« Exhibit infinite viscosities + Non-uniform velocity distribution when flowing, + Compressible + Experience friction and turbulence in flow 2 SHAPTERONE FLUID MECHANICS Properties of Fluids & HYDRAULICS Real fluids are further divided into Newtonian fluids and non-Newtonian fluids Most fluid problems assume real fluids with Newtonian characteristics for convenience, This assumption is appropriate for water, ir, gases, steam, and other simple fluids like alcohol, gasoline, acid solutions, etc, However, slurries, pastes, gels, suspensions may not behave according to simple fluid relationships Fis) Eon" Se) ba CS anna Tentorion Fs) [Reefiewtonin Fa een Fide Figure 1 - 1: Types of ud ([ingram uae) Dalatant Fd MASS DENSITY, p (RHO) The density of a fluid i its mass per unit of volume, CHAPTERONE 3 ee sancHANCS Properties of Fluids & HYDRAULICS: where: p= absolute pressure of gas in Pa R= gas constant Joule / kg-°K “Table 1 4: Approximate Room-Temperature Densities of Common Flids pinkg/m 729) | Alcohol Ammonia ‘Glycerin, Stn 13,600 Water 7,000 SPeciric VOLUME, Vv, Specific volume, V, is the volume occupied by a unit mass of fluid. Eq.1-3 ‘mas of uid ae volume, V vane a 2 Units English = sugs/f® Note pan ~ Se peek eentann UNIT WEIGHT OR SPECIFTE WEIGHT, st kg/m? Gpecific weight or unit weight, 7 is the weight of a unit volume of a fluid. ‘or an ideal gas, its density can be found from the specific gas constant and. ideal gas law: ads 2 i ae Fqg.1-2 fluid, W qi 12a weight 4 CHAPTER ONE FLUID MECHANICS FLUID MECHANICS CHAPTERONE Properties of Fluids E HYDRAULICS & HYDRAULICS Properties of Fuids Ue the upper plate will adhere to it and will move with the same velocity U while English: Ib/st® the fluc in contact with the fixed plate will have a zero velocity. For small Metric: dyne/em* values of U and y, the velocity gradient can be assumed to be a straight line SI Nfos' or KN/m? and F varies as A, U and y as bau we a Sipeum ty Se ie wut Ye AY (fromthe figure)“ Specific gravity, », ia @ dimensionless saliv of a fluid’s density to some yay an standard reference density. For liquids and solids, the reference density is nie paar a) = Shearing stress, Phguia gee be eon > absolute viscosity denoted as u. In gases, the standard reference to calculate the specific gravity is the derisity dV ofalr. 7% Pane Eq.1-8 so See Eq.1-7 4 Pat : Tor water at °C: ° y= 624 Ib/f = 9.81 KN/m? 94 slugs/f = 1000 kg/m? 10 VISCOSITY, 1 (MU) The property of a fluid which determines the amount of its resistance to shearing forces. A perfect fluid would have no viscosity Consider two large, parallel plates at a small distance y apart, the space between them being filled with a fluid Consider the upper plate to be subject to a force F so as to ‘move with a constant velocity U, The fluid in contact with ‘hed ite shear stress in Ib/f€ or Pa = absolute viscosity in lb sec/ ft (poises) or Pa-soc y= distance between the plates in ft or m ocity in ft/s or m/s KINEMATIC VISCOSITY y (NU) Kinematic viscosity is the ratio of the dynamie viscosity of the fluid, 4, to its mass density, o. where = absolute viscost y 1m Fa-see p= density in kg/m FLUID MECHANICS 6 SHAPTERONE & HYDRAULICS Properties of Fluids FLUID MECHANICS CHAPTERONE = S HYDRAULICS, Properties of Fluids Table 1 - 2: Common Units of Viscosity system Absolite, Kinematic, v ae, Thsec/fe Fy English Oneie £8/sec cdynes/cm? ome Metric y | (poise) (stoke) | Pas F EF (N-s/m’) ee Note: 1 poise = 1 dynes/em 1 stoke = 0,001 m/s O41 Passéc (1. dyne= 105 N) SURFACE TENSION o (SIGMA) The membrane of “skin” that seems to form on the free surface of a fluid is due to the intermolecular cohesive forces, and is known as suoface tension. ‘Surface tension is the reason that insects are able to sit on water and a needle is, able to float on it. Surface tension also causes bubbles and droplets to take on a spherical shape, since any other shape would have more surface area per unit volume. Pressure inside a Droplet of Liquid: where: = surface tension in N/m diameter of the droplet in.m gage pressure in Pa Capiltarity (@) Adhesion > cohesion (©) Cohesion > adhesion Chpillarity (Capilary action) is the name g thin-bore tube. The rise or fall or a fluid in a capil surface tension and depends on the relative magnitudes of the cohesion of the liquid and the adhesion of the liquid to the walls of the containing vessel. Liquids rise in tubes they wet (adhesion > cohesion) and fall in tubes they do Hot wet (cohesion > adhesion). Capillary is important when using tubes smnaller than about 3/8 inch (9.5 mm) in diameter _ deeos0 vd iven to the behavior of the liquid in a tube is caused by h wean | For complete wetting, as with water on clean glass, the angle @ is 0° Hence the formula becomes ) win | ‘k= capillary rise or depression in m ‘y= unit weight in N/m? diameter of the tube inm = surface tonsion in Pa CHAPTER ONE 8 FLUID MECHANICS FLUID MECHANICS CHAPTERONE —«g) & HYDRAULICS Properties of Fluids Properties of Fluids & HYDRAULICS Table 1 3: Contact Angles, 0 Materials ‘Angle, 0 mereury-plass 1 Water-paraffin 107 water-silver | Kerosene-glass [elycerin-glass ‘ethylalcohol glass COMPRESSIBILITY, Compressbiity (also known as the coefficient of compressibility) is the fractional change in the volume of a fluid per unit change in pressure in a constant ~ aress | Ap ¥g.1-15 stain AV PRESSURE DISTURBANCES Pressure disturbances imposed on a fluid move in waves. The velocity or felerity of pressure wave (also known as acoustical or sonic velocity) is expressed as: o= [fa = Jo Eq.1-17 Temperature process EE AW a p= te Bq.1-18 - bin PROPERTY CHANGES IN IDEAL GAS For any ideal gas experiencing any process, the equation of state is given by’ orp at Bq t- 14 z where: AV = change in volume V = original volume Ap change in pressure V/V = change in volume (usually in percent) BULK MODULUS OF ELASTICITY, Ey The bulk modulus of elasticity of the fluid expresses the compressibility of the fluid. It is the ratio of the change in unit pressure to the corresponding volume change per unit of volume, Eq.1-18 When temperature is held constant, Eq, 1-18 reduces to (Boyle's Lino) pM Eq.1-19 | When temperature is held constant (isothern 18 reduc to (Charle’s Lew) , CHAPTER ONE 10 _Propertios of Fuias FLUID MECHANICS & HYDRAULICS, ‘FLUID MEcHanics CHAPTERONE 44 HYDRAULICS Properties of Fluids For Adiabatic or Isentropic Conditions (no heat exchanged) pe Vika pa Va Bg.1-21 or = 22 =Constant Eg. 1-22 Ph aud By 1-23) initial absolute pressure of gas final absolute pressure of gas Vi= initial volume of gas /2= final volume of gas T= initial absolute temperature of gas in °K (°K Tz final absolute temperature of gas in °K k = ratio of the specific heat at constant pressure to the specific heat at constant volume, Also known as adiabatic exponent. +273) VAPOR PRESSURE Molecular activity in a liquid will allow some of the molecules to escape the liquid surface. Molecules of the vapor also condense back into the liquid. The vaporization and condensation at constant temperature are equilibrium processes, The equilibrium pressure exerted by these free molecules is known as the mpor pressure or saturation pressure Some liquids, such as propane, butane, ammonia, and Freon, have significant vapor pressure at normal temperatures, Liquids near their boiling point or that vaporizes easly are said to volalile liquids, Other liquids such as mercury, have insignificant vapor pressures at the same temperature. Liquids with low vapor pressure are used in accurate barometers The tendency toward vaporization is dependent on the temperature of the liquid, Boiling occurs when the liquid temperature is increased to the point hat the vapor presstite is equal ta the lncal ambient (sisrraunding) pressive. ‘Thus, a liquid’s boiling temperature depends on the local ambient pressure, as well as the liquids tendency to vaporize ‘Table 1 4: Typical Vapor Pressures Fluid KPa, 20°C, mercur @.000173, ‘turpentine 0.0534 ‘ethyl alcohol ther Butane Freon Propane. Solved Problems __ aa ey al Problem 1-1 IA reservoir of glycerin has a. mass of 1,200 kg and a volume of 0.952 cu. m Pind its (a) weight, W, (0) unit weight. 7, (&) mass density, p, and (4) specific “gravity (9 olution (a) Weight W= Mg 1,200)(9.81) = 11,772 N of 11.72 kN Weight, W (Unie weight, y= © Density, p 260.5 kg/m? 12. CHAPTER one e FLUID MECHANICS erties of Fluids & HYDRAULICS Spedicgevtiy« ey Pwater 1605 Specie gravity,» = 1280 Specific gravity, s = 1.26 Problem 1-2 Me Pe ea ot The specific gravity of certain oil is 0.82. Calculate its (a) specific weight, in 1/#€ and KN/am’, and (3) mass density in slugs/fO and kg/m’. Solution (@) Specific weight, y Specific weight, Specific weight, 81 08 2 = 8.044 kN/m? (6) Density, p= Puan § Density, p = 1.94 0,82 = 1.59 sluge/ft® Density, p = 1000 x 0.82 = 820 kg/m? CON Siar. eee ene ee aoe ae A liter of water weighs about 9.75 N. Compute its mass in kilograms. Sol Mass = s 9. Mass = 22 981 Mass = 0.994 kg, Problem 1-4 Ian object has a mass of 22 kg at sea level, (2) what will be its weight at a point where the acceleration due to gravity = 9.75 m/s?? (b) What wil be its ‘mass at that point? FLUID MecHANics CHAPTERONE 4.3 HYDRAULICS. Properties of Fluids Solution () W=mg=22(9:75) v 2145 N (6) Since the mass of an object is absolute, its mass will still Problem 1-5 What is the weight of a 45-kg boulder if it is brought to a place where the Acceleration due to gravity is 395 m/s per minute? Solution m/s, Imin 395 2/8 ‘min * 60sec 6.583 m/s? W=45(6.583) W= 296.25 N lem 1-6 Mi spect: volume of « certain gas ts 07849 s02/ g, whit is ils specific ~ o7et8 kg/m, Specific weight, y= ps 1 =125 Nim Specific weight, CHAPTER ONE 14 Properties of Fiulds FLUID MECHANICS & HYDRAULICS Problem 1-7 ‘What is the specific weight of air at 480 kPa absolute and 21°C? Solution =pxg p iin > where R = 287 J/kg-°K aoe I kg _480%10° 21+273) p= 5.689 kg, 1 = 5.689 9.81 55.81 N/m? Problem 1-8 Find the mass density of helium at a temperature of 4 °C and a pressure of 184 kPa gage, if atmospheric pressure is 101.92 kPa. (R = 2079 J/kg * °K) Solution Density, p= P= Prog * Pa = 181 + 101.92 2 kPa 273= 277K 285.92 10 2,079(77) Density, p = 0. 4965 kg/m? Density, p Problem 1-9 \t32C and 205 kPa gage, the specific weight of a certain gas was 13.7 N/m. determine the gas constant of this gas. FLUID MECHANICS CHAPTER ONE & HYDRAULICS. Properties of Fluids 15 Solution Density, p = 1.397 kg/m Density, p= = RT (205-+ 101.325) «10° ea ee) et Note: Pan 32 +273) 718.87 kg °K Gas constant, R 101.225 kPa Problem 1-10 Air is kept at a pressure of 200 kPa absolute and a temperature Sp0-Liter container, What is the mass of alt? £ 30°C in Solution 287(80+ 2 23 kg/m? Mass = px V Mass = 1.15 kg Problem 1-11 A cylindrical tank 80 em in diameter and 90 cm high is filled with a liquid. The tank and the liquid weighed 420 kg. The weight of the empty tank is 40 kg. What is the unit weight of the liquid in kN/m. 16 L=0.192m=192em Problem i~i3 A liquid compressed in a container has a volume of 1 iter at a pressure of 1 MPa and a volume of 0.995 liter at a pressure of 2 MPa. The bulk modulus of elasticity (Ep) of the liquid is: Solution ap i W/V 035-171 00 MPa Ey Problem 1-14 What pressure is required to reduce the volume of water by 0.6 percent? Bulk modulus of elasticity of water, Es = 2.2 GPa, | FLUID MECHANICS CHAPTERONE 4. & HYDRAULICS Properties of Fluids Solution Es Do06v /V p= 0.0132GPa p= 13.2MPa Problem 1-15 7 Water in a hydraulic press, initially at 137 kPa absolute, is subjected to a pressure of 116280 kPa absolute. Using Fe = 25 GPa, determine the percentage decrease in the volume of water. Solution at dp av IV 116,280=137) 10° sxips a es aviv V = 9.0465 4.65% decrease sa v a v Problem 1-16 [f 9 my of an ideal gas at 24 °C and 150 kPaa is compressed to 2 m?, (a) what is the resulting pressure assuming isothermal conditions. (b) What would hav is isentropic. Use k= 1.3, been the pressure and temperature ifthe prt CHAPTER ONE 18 properties of Fuids (@) For isothermal condition PVi= pa Ve 1150(9) = p> 2) p2= 675 kPa abs () For isentropic process PiVat= pe Va! 150(9)'*= ps @)* 2 = 1,060 kPa abs fy, ene Tr (pa Ty ly T_ (1.060)020718 +73 \ 150 Tz = 466.4°K or 193.4°C ease ee a ere nL 1 PE ec eet Problem 1-17 If the viscosity of water at 70 °C is 0.00402 poise and its specific gravity is 0.978. determine its absolute viscosity in Pa - s and its kinematic viscosity in m2/s and in stokes, Solution Absolute viscosity 00402 poise x 94Fa=s “poise X y= 0.000802 Pa -s Kinematic viscosity 1 _ 0.000402 (100% 0.978) 11 x 107 m/s Y= 4.11 x 10? mi/s x — Ke v= 4.11 x 109 stoke S001: 76 R ONE cHAPTEI 19 FLUID MECHANICS Properties of Fluld roperties of Flulds & HYDRAULICS Mvo lage plane surfaces are 25:mm apart and the space between them filled Milt a tiguic of viscosity = 0.958 Pas, Assuming the velocity gradient tobe fragt line, what force ts required to pull avery thin plate of 037 me area at Bonstant speed of 0.3 m/f the plate s 84 mum from one of the surfaces? Solution ie | _F/A 25 mm | Hu usy a4 =p WA atl 958 3 95803037) 6 4 0.0166 0.95803)037) _|, Fa 008s - 66N 64+ 12.66 9.06 N J oylinder of 125 mm radius rotates concentrically inside a fixed cylinder of WW mm radius. Both cylinders are 300 mm long. Determine the viscosity of the liquid which fills the space between the cylinders if a torque of 0.88 N-m is equired to maintain an angular velocity of 2m radians/sec Assume the a straight line Velocity gradient t CHAPTER ONE 20 FLUID MECHANICS Properties of Fluids & HYDRAULICS 125(2n) sexing 785 m/s aie y= 0.005 m Torque = F(0.125) Torque = A (0.125) bbe stat 0.88 = [2n(0.125(03)] (0125) +=2988 Pa 2988 0785 /0.005 = 0.9 Pa-s ons -f Problem 1-20 An 18-kg slab slides down a 15° inclined plane on a 3-mm-thick film of oil with viscosity = 0.0814 Pa-sec. If the contact area is 0.3 m®, find the terminal velocity of the slab, Neglect air resistance, Solution We 169.8) = 176.5801 Terminal velocity is attained when the motion is zero sum of all forces, in the direction of FLUID MECHANICS CHAPTERONE 4 & HYDRAULICS Properties of Fluids [SF =O] Wsin®-F.=0 F.=Wsind F,= 176.58 sin 15° 17658 sin 15° = 0 0814 14 m/s = 5.614 mys Problem 1-24 timate’ the height to which water will rise in a capillary tube of diameter 3 ter Solution Note: 0 = 90° for water in clean tube 40 Capillary rise, 140.0728) 7310(0.003) Capillary rise, r= 0.0099 m = 9.9 mm Capiltfry vise, b= Problem1-22 Estimate the capillary depression for mercury in a glass capillary tube 2 mm in diameter, Use ¢ = 0.514 N/m and 8 = 140" Solution 4ocosd _ _4(0.514)(cos 140°) YH (9810+ 13.6)(0.002) Capillary rise, Capillary rise, negative sign indicates capillary depression Capillary depression, h = 5.9 mm CHAPTER ONE FLUID MECHANICS ID MECHANICS 22 _ Properties of Fluids & HYDRAULICS Problem 1 - 23 What is the value of the surface tension of a small drop of water 0.3 mm in diameter which is in contact with air if the pressure within the droplet is 561 Pa? blem 1 - 26 iween impulses, how deep is the water? 6 a & Yelocity of the pressure wave (sound wave) is ee a ms Vp oN oat oe - 428 m/s 6 000 a Problem 1-24 the echo is received 8 ilway between impulses, then < An atomizer forms water droplets 45 um in diameter. Determine the excess es te ° Pressure within these droplets using o = 0.0712. N/m, vs sec and the total lance covered is 2i, then: Solution ns Fa dhect i a= 128% MOO7I2) 6 05 na We 1785m x10 lem 1 - 27 What pressure will 80 °C water boil? Wopor pressure of water at 80°C = 474 kPa Problem 1-25 Distilled water stands in a glass tube of 9 mm diameter at a height of 24 mm. What is the true static height? Use c = 0.0742 N/m. olution Solution Water will boit i the atmospheric pressure equals the v _ secos eas Therefore water at §0 °C will boil at 47.4 kPs where 0 = 0° for water in glass tube n= OAD) 4.00336 m = 3.36 mm 3810(0.000) True static height ~ 24 -3.36 ‘True static height = 20,64 mm er accond tf the deve held Paurace of es wrter (x= 20810 a) and he eco i receved mwa CHAPTER ONE Properties ufFuids 23 Jsoura wave charren one 24 Properties of Fluids Supplementary Problems Problem 1 - 28 What would be the weight of 1 3-kg mass on a plane! due to gravity is 10 m/s FLUID MECHANICS CHAPTERONE > & HYDRAULICS, tt gt Properties of Fluids & HYDRAULICS Fee pales es ool of nitrogen at 30° and 125 KPa ab is peri Dotermally o 30 mf what ste sealing prerere?() What would the wwe been if the process had been isentropic? Jpressuire and temperature have been ifthe pi Lea (8) 34.7 kPa abs; -63°C t where the acceleration Ans: 30 N Problem 1-29 A vertical cylindrical tank with a diameter of 12 m and a depth of 4 m is filled with water to the top with water at 20°C. If the water is heated to 50°C, how. auch water will spill over? Unit weight of water at 20°C and 80°C 1g 979 KN/m? and 9.69 kN/m, respectively, Boe us sage slides down an A squre block weighing 1.1 KN and 250 mm on an edge slides down an line on im of 60 hee Assuming nen veloc pron the fil and neglecting air resistance, what is the terminal velocity of the block? *The viscosity of cil is7 mPa-s. Angle of inclination is 20" Ans: 5.16 m/s Ans: 4.7 mi Problem 1 - 30 A Higid steel container is partially filled witha liquid at 15 atm, The volume of Pesnittid 8 123200 L. At a pressure of 30 atm, the volume of the liquid is 2.231001. Find the average bulk modulus of elasticity ofthe liquid over the given range of pressure if the temperature after compression is allowed to ‘eturn to its inital value, What isthe coefficient of compressibility? Ans: En = 1.872 GPa; P - 0.534 GPa" Problem 1-35 Benzene at 20°C has a viscosity of 0.000551 Pa-s. What shear stress is required 1o deform this fluid at a strain rate of 4900 5"? ena Problem 1-36 A shaft 70 mm in ciameteris being pushed a speed of 400 mm/s through a Hearing sleeve 702 mm in diameter and 250 min long, The clearance, assumed niform, is filled with oil at 20°C with v = 0.005 m’/s and sp. gr. = 0.9. Fin« the force exerted by the ol in the shaft Problem 1-31 Cateulate the density of water vapor at 350 kPa abs'and 20°C if ts gas constant 4s 0.462 kPa-m'/kg.°K a. Ans: 2.59 kg/m! Problem 1 - 37 ‘wo clean parle glass plate, separated bya distance = 1.5 may are dipped {n.a bath of water. How far does the water rise due to capillary action, if ¢ 0730N/m? Problem 1-32 Air is Kept at a pressure of 200 kPa and a temperature of 30°C in a SO0-L container. What is the mass of the air? Ans: 9.94 mm Ans: 1.15 kg, 26 Specie Grovty blem 2-2 (2) Open manometer (©) iferentiat manometer itis the pressure 12.5 m below the ocean? Use sp. gr. = 1.08 for salt water. lution poyk p= (81 »1.05)0125) 1263 kPa (©) Prezometer 36 CHAPTER TWO FLUID MECHANICS ial eee ieee Principles of Hydrostatics & HYDRAULICS Principles of Hydrostatics 37 Problem 2-3 Problem 2-5 If the pressure 23 meter below a liquid is 388445 kPa, determine its uni If the pressure in the air space above an oil (= 0.75) surface in a closed tank is weight y, mass density p . and specific gravity s 115 kPa absolute, what is the gage pressure 2 m below the surface? Solution Solution (a) Unit weight. y payh 338.415 — y 23) Y= 14.715 Ni P= Poutuce +Y Prune = 115 ~ 10125 Note: Pan = 101.325 kPa Put = 13.675 kPa gage p= 13.675 + (9.81%0.75)(2) 28.39 kPa 18) Mass density» pet i hiiaticha aii g Problem 2-6 14715010" Find the absolute pressure in KPa ata depth of 10 m below the fre surface of 98 il of sp. gr. 075 ifthe barometric reading is 752 mmHg = 1,500 kg/m a (0) Specific gravity, ae = Pets Pom = Yale fae (81 = 13.6)0752) 1.500 pm ~ 100229 kPa Wate Prats "00.329 + (981 « 0.7>\(10) oaye Pots = 173.9 kPa Problem 2-4 a Problem 2-7 If the pressure at a point in the ocean is 60 kPa, what is the pressure 27 meters AA presstire gage 6 m above the bottom of the tank containing a liquid reads 90 below this point? kPa. Another gage height 4 m reads 103 kPa, Determine the specific weight of the liquid Solution The difference in pressure between any two points in a Solution liquid is po pi = yh poo pit yh = 60 + (9.81%1.03)(27) p= 332.82 kPa popayh 103 - 90 = y(2) y= 65 kN/m® CHAPTER TWO 38 Principles of Hyarostaties Problem 2-8 [An open tank contains 5.8 m of water covered with 3.2 m of kerosene (y = KN/m), Find the pressure at the interface and at the bottom of the tank. FLUID MECHANICS & HYDRAULICS Solution (a), Pressure atthe interface ea = 02) a= 256 kPa Kerosene es (0) Pressure at the bottom Sem? m penta ‘A eet oli sia 5815.8) + 862) a= 82.498 kPa 2 Problem 2-9 If atmospheric pressure is 95.7 kPa and the gage attached to the tank reads 188 mmbig vacuum, find the absolute pressure within the tank. Solution Pas = Pa + Per Prog ™ Yr Herry (981 x 13.6)(0.188) (08 kPa vacuum Paras * 25.08 kPa paw = 95.7 + (-25.08) ats = 70.62 kPa abs Problem 2-10 ‘The weight density of a mud is given by y= 10+ 0.5h, where yis in kN/m? and his in meters, Determine the pressure, in kPa, at a depth of 5 m CHAPTER TWO Principles of Hydrostatics FLUID MECHANICS & HYDRAULICS Solution Since the density of the mud varies with depth, the pressure should be solved by integration ap=y dk p= (10+ 05 hth p= 10h+025H" | (106) + 0.256) -0 56.25 kPa blem 2= i Ui the figure shown ifthe atmospheric Pressure is 10103 kPa and the absolute tre atthe bottom of the tank is kPa, what isthe specific gravity sive oil” SAE OI, s= 0.89 Ove, s =? Mercury, $ = 13.6 lution Goge pressure at the bottom of the tank, p = 231.3 ~ 101.03 Gage pressure at the bottom of the tank, p = 130.27 kPa [p= 2a P= Yu lta Yo e+ Yo lh + Yo to 13U2/ = (BBL « 15,6)(04) + 9.81 38 39 592.9) + 98125) | 0.81 « 089/15) CHAPTERTWO gy CHAPTER TWO. FLUID MECHANICS FLUID MECHANICS BE raewcsrtics 40 _Frinciples of yarostatcs & HYDRAULICS |) _& HYDRAULICS Bieles of Hy Female Pooeaer ide of 1,200 m if the If air had a constant specific weight of 122 N/m? and were incompressible Compute the barometric pressure in KPa at an altitude, of 1.200 e : & e Pa. As {isothermal conditions a 21°C. Use what would be the height ofthe atmosphere if the atmospheric pressure (seal pressure at sea Tove s 101.3 Pa, Assume level) is 102 kPa? R= 287 Joule /kg-*K. Solution Solution Helght of atmosphere, h= 2 Hyves 10210" pa Height of atmosphere, = = 8,360.66 m Problem 2 43 (CE Board May 1994) = 000001185 p Assuming specific weight of alr to be constant at 12 N/m, what is the approximate height of Mount Banahaw if a mercury barometer atthe base of 4p =-(0.00001185 (981) dt the mountain reads 654 mm and at the same instant, another barometer at the 0001168 dt top of the mountain reads 480 mm, aa | antes fam a0 3 , 20 inp = 0,0001163% Psa I, Inp= In (101.3 105 = -0.000116(1200 - 0) Inp= 11386 286 pre p= 88,080 Pa Pros Pop = 1 (tw Hs) ection ~ (Yu ta)op = Ct Hae {9810 x 13.6)(0.654) - (9,810 x 13.6)(048) = 12 f= 1,934.53 m chapter TWO CHAPTER TWO FLUID MECHANICS rostatics 43 42 _Principies of Hyarostatics & HYDRAULICS Principles of Hydrostat Problem 2-15 blem 2 - 18 (CE November 1998) le that of piston B is 950 69. cm. Convert 760 mim of mereury to (a ol of sp. gr 0.82 and (@) water, on A has a cross-section of 1200 sq. cm while that of pisto fe wae th the latter higher than piston A by 1.75 m, Ifthe intervening passages are Solution led with oil whose specific gravity is 08, what is the difference in pressure 0) a= 136 = 0763s oe 9,810 x 08)(1.75) hh 12.605 m of ol Beaman ©) Tate * Hers Sery 0.76(13.6) Fisate = 10.34.m of water Problem 2-16 (CE Board May 1994) A barometer reads 760 mmFig andl a pressure gage attached to a tank reads £850.cm of ol (sp. gr. 0.80), What is the absolute pressure in the tank in kPa? lem 2-19 the figure shown, Solution ine the weight W Pas™ Pam + Pe can be cartied by the ~ (9.81 « 13.1076) + 0.81 x0.8)8.5) EN force Acting, an the Pony = 168.1 kPa abs a Of, s= 082 Problem 2- 17 A hydraulic press is used to raise an 80-KN cargo truck. If oil of sp. gr. 082 mo acts on the piston under pressure of 10 MPa, what diameter of piston is required? olution Solution Since points 1 and 2 ie on the Sneha Since the pressure under the piston is uniform: fame elevation, ps= ps oa Force = pressure x Area £80,000 = (10x 109) # D2 D=01m=100mm » W=150kN CHAPTER TWO Principles of Hydrostatics 44 Problem 2-20 A drum 700 mm in diameter and filled with water has a vertical pipe, 20 mi in diameter, attached to the top. How many Newtons of water must poured into the pipe to exert a force of 6500 N on the top of the drum? a Zs 56 Abe bark 16,904 = 9810 i: h=1.723m Weight = yx Volume = 9810 $ (0.02)%(1,723) ‘rea on top Weight=5.31N 700 mim Problem 2-21 cylinder. What force F is required to balance the weight of the cylinder ifthe weight of the plunger is negligible? Poe, Ja 0.00823 mt joi, + -0.70 Jol, s = 0.78 CHAPTER TWO t NT Principles of Hydrostatics & HYDRAULICS 45 +a lcpraee nate Niet an Ja = 0.305 \Eso129 w Noi, s = 0.78 You, s = 0.78 ty 00303 pr=309.6 F (Pa) Wir ous Pan” o3m po 13622 KPa 136.22 - 309.6 F = (9.81 « 078)(4.6) F=0.326 kN = 326 N Problem 2-22 The hydraulic press shown is filled with oil with sp. gr. 0.82. Neglecting the weight of the two pistons, what force F on the handle is required to support the TO KN weight? CHAPTER TWO Principles of Hydrostatics 47 46 SHAPTERTWo FLUID MECHANICS Principles of Hydrostatics & HYDRAULICS [LUID MECHANICS & HYDRAULICS Solution then the reading is equivalent to'30 cm of gasoline [ince the gage reads “FULL” Since points | and 2 lie on the same elevation, then, map — song 400mm Reading (pressure head) when the tank contain iy + 2h) em of gasoline y+2qly =30 y= 27.06 em Ca (0.425) = 1.11(0.025) 25mm 000m F = 0.0654 kN Peet =654N ‘Problem 2 24 (CE Board November 2000) 4m Determine the value [= Mo = 0} F(0.425) = F(0.025) lor the tank shown in the Figure, fy = 3m and Ih of Fe FB0 of the lever arm Problem 2- 25 Solution Summing-up pressure head from 1 to 3 in meters of water 5 ngogay-x= PS Y Y The fuel gage for a gasoline (sp. gr. = 0.68) tank in a car reads proportional to its bottom gage. If the tank is 30 cm deep an accidentally contaminated with 2 ‘em of water, how many centimeters of gasoline does the tank actually contain when the gage erroneously reads “FULL”? ae 0+ 0.84 I~ (4-3)=0 ia fy = 119m Tr sagt oen| _Gablne, § = 068 roa Ly Ful CHAPTER TWO Principles of Hydrostatics 48 ae CHAPTER TOTO oes Priciptot of Fyarostaties Problem 2 - 25 (CE Board May 1992) |m the figure shown, what is the static pressure in kPa in the air chamber? joMt bso 39) {S$ ~§ mgidov9 Problem 2 - 26 (1008 rod For the manometer shown! rivarle vtornonn determine the pressure at the a center of the pipe eran, = = 1355 Solution The pressure in the air space Solution equals the pressure on the surface Sum-up pressure headt from of oll. px 103 in meters of water: noo athe 98102) p= 19.62 kPa Pa pom tol £14758) 1962 ps = (981 « 0.80)(a) py= 148.7 kPa p= 177 kPa Another solution Sum-up pressure head from 1 to3 in meters of water Pr 42 40.80) = 22 422326 2 CHAPTER TW 50 P. Principles of Hydrostaties Problem 2-27 (CE Board November 2001) Determine the value of yin the manometer shown in the Figure, im BE Solution Summing-up pressure head from A to B in meters of water PA 4908) 415-9136) 5 Ps sor +39- 13.6y 981 aes where pe = 0 y= 0.324 m FLUID MECHANICS Mf FLUID MECHANICS fesctiArTeR TWO S& HYDRAULICS I & HYDRAULICS Principles of Hy a im f si Mercury Problem 2-28 (CE May 1993) jo 30am | In the figure shown, when the a! funnel is empty the water surface is at point A and the mercury of sp. gt. 1355 shows a deflection of 15 cm. Determine the new deflection of mercury when the fonnel is filled with water to B, Solution figure (a): Level at A Figure (b): Level at 8 Solve for y in Figure (0): ‘Sum-up pressure head from A to 2in meters of water: Pa + y-015(1388) = 2 O+y-203=0 y= 203m 52 CHAPTERTWO FLUID MECHAR De anics CHAPTERTWO 53 Principles of Hydrostatics & HYDRAULICS DRAULICS Principles of Hydrostatics In Figure (b) When the funnel is filled with water to B, point 1 will move down to with the same value as point 2 moving up to 2” Bp preseare head froma 20 eters of water % yig-2= Be 5 7 136y- In Figure @): ‘Sum-up pressure head from B to 2! CMa: Py Y ae ty tre (x +0154 2)(13.5 op O84 yt x= (6+ 015 + 31355) Bi. 1p pressure head from 2 to mt in meters of water: Pa 0+080+203+x-271x-2m=0 Bo + @asino+y+02)09.6)- (+02) ~ Pat 1 261x=080 ¥=0.031m=3.1em 0+ 272sin0 +13.6y+2.72-%-02 1B6y-x=8183-272sin0 Eq. (2) New reading, R=15 + 2x =15+2G.1) New reading, R = 21.2 em Be [136y -»=13.6y-3) 8.183 -2.72sin0 = sin 9 = 03852 0-22.66" m0 Problem 2 - 29) ‘The pressure ut point m in the figure shown was increased from 70 kPa to 105 kPa. This causes the top level of mercury to move 20 mm in the sloping tube. What is the inclination, 6? lem 2-30 flosed cylindrical tank contains 2.m of water, 3 m of oil (s= 0.82) and the air col has a pressure of 30 kPa. If an open mercury manometer at the of the tank has 1 m of water. determine the deflection of mercury. Solution tion ‘Sum-up pressure head from Lto 4 in meters of water: ae +3(0.82) +2+1 - (13.6) = Bh +246 +3-13.6y=0 y= 0.626 m Figure (a) Figure (b) In Figure (a) CHAPTER TWO 54 Principles of Hyarostatics Problem 2- 31 The Uctube shown is 10 mm in diameter and contains mercury. [f 12 ml of water is poured into the right-hand leg, what are the ultimate heights in the two legs? ae L Solution Solving for hi, (ee figure b) Volume of water = Law = 15.28 em Since the quantity of mercury before and after water is poured remain the same, then; 1208)=R+3 412044 Re2x=240 > Bq. (1) 20K — 1 20mm —! 1 20mm Figure (a) Figure (b) CHAPTER TWO Principtes of Hydrostatics 95 In Figure () ‘Summing-up pressure head from 1 to 3 in mm of water: Pt + 1528- RG3.6) = 22 7 7 R=1124mm InEq. 2 11.24 +2 x1 Ultimate heights in each leg Right-hand leg, n= h+ x =1528 + 114.38 Right-hand leg, fy = 267.18 mm_ Left-hand leg, n= R+ x 11.244 114.38 Left-hand leg, Jy «125.62 mm jem 2-32 ‘ gage reading of -17.1 kPa, fine the (f) rlevations of liquids in the open weometer columns E, F, and and (b) the deflection of the Mercury in the U-tube Manometer neglecting the ‘weight of air cHaprer two 56 Principies of Hyarostatics Solution FLUID MECHAM & HYDRAULI { ane a Column £ Sum-up pressure head from 1 toe in metes of water, Ls iyor) = 2° Y y He + incor hy =25 m Surface elevation Surface elevation 15-hy 15-2. 25m Column F Sum-up pressure head from 1 to fin meters of water exon mn}= #2 ae Surface elevation 12+ 0.357 = 12.357 m FLUID MECHANICS CHAPTER TWO ‘& HYDRAULICS 57 Principles of Hydrostatics Column G ‘Sum-up pressure head from 1 tog in meters of water: PL 4300.7) +4()-In.6) * SUL 421 +4-16hu=0 In=272m Surface elevation = 8 + ip Surface elevation =6 1 2.72-10.72-m Deflection of mercury ‘Gum-up pressure head from 1 to5 in meters of water; PL 4300.7) +4+4-Iu(13.6) 7 2H +101-13.6% y= 0.614 m roblem 2-33 ‘An open manometer attached to a pipe shows a deflection of 150 mung with the lawer level of mercury 450 mm below the centerline of the pipe carrying, water, Calculats the pressure at the centerline of the Pipe Sum-up pressure head from 1 to Hin meters of water; Ph 4 o45-0.15(13.6) = 2 Y 7 PL +045-204=0 38 pin 15.6kPa CHAPTER TWO 58 Principles of Hydrostaties Problem 2-34 For the configuration shown, calculate the weight of the piston if the pressure gage reading is 70 kPa, CHAPTERTWO gg FLUID MECHANG Principles of Hydrostatics ILUID MECHANICS HYDRAULICS )) Gage liquid = mercury, = 0:1. m ‘Sum-up pressure head from 1 to 4 in meters of water; Ph ype be MI3)-¥-15" Y b Ba Y 5-01 + 0.10136) 76 m of water carbon tetrachloride Weight Sum-up pressure head from 1 to 4in meters of water; PL ath h(L58)-1-15= Bs 5 7 Solution Sum-up pressure head from Ato Bin meters of water; imo 2A 140.86) = 28 1 "Piston ee oi 52086 Weight = Fr pax Area = 78.44% 50)? Weight = 61.61 kN ‘Problem 2- 35 Two vessels are connected to a differential manometer using mercury, # connecting tubing being filled with water. The higher pressure vessel is 1.5 lower in elevation than the other. (a) If the mercury reading is 100 mm, wh is the pressure head difference in meters of water? (2) If carbon tetrachloride 1.59) were used instead of mercury, what would be the manomete ( reading for the same pressure difference? Bi Ps 154059 . where 21. P& =2.76 m > from (a) BR oy 276=15+ 059% he=2.136m blem 2-36 In the figure shown, determine He height i of water and the ype reading at A when the lute pressure at B is 290 ka cHapTer TWO 60 starrer nwo FLUID MECHANIGg) FLUID MECHANICS eee Principles of Hydrostaties & HYDRAULIES| | & HYDRAULICS selbtion Sumn-up pressure (gage) head from 1 tod in meters of water: FL + x(0.9) +130.) -1.3(13.6)= cd Y ‘Sum-up absolute pressure head from B to 2 in meters of water; Bb ee AP + 09x-1651-0 381 x=1381m 8 0.7036) - i 700 rom ‘Then, x+ y= 2842 m 1 >o 6 ols $= 085 Problem 2-38 For the manometer setup determine the difference in pressure between A and B. Sum-up absolute pressure head from B to A in meters of water; 28 97036) +07= 2A Y Y shown, FR -952407= 2A 81 9 = 2035 kPa abs Problem 2-37 In the figure shown, the atmospheric ea Solution pressure is 101 kPa, the gage A +068ny+17 reading at A is 40 kPa, and the vapor oa T x-y~102m * > Fq (1) pressure of alcohol is 12 KPa | absolute. Compute x+y 1 SSum-up pressure head from A to B uf inmeters of water; , a3 Pax 0680085) + y= FE tary Pa PB ax.ys 0578 > Fa.) Solution = 124908 Substitute x - y = 1.02 in Eq, (1) to Eq. 2}: ‘Sum-up absolute pressure head from ae eee Saar ~ 102i Ba. 1 to2in meters of water, Pa _ Pa -102+0578 et 2 -yos) Pa=Ps = 1508 404101 12 a See pa po=15.68kPa 981 Yost y=1461m CHAPTER TWO Principles of Hydrostatics FLUID MECHANICS 62 Problem 2-39 A differential manometer i attached to a pipe as shown, the difference between points Caleulate pressun and B. Solution Stim-up pressure head from A to B in meters of water bs (0.9) -0.1(13.6) + 0.1(0:9) + (0.9) = 0.1(13.6)-0.1(0.9) Pap 981 pe= 1246 kPa 5 Solution CHAPTER TWO Principles of Hydrostatics 63 the figure shown, the 0.25 m lection of mercury is initially mmm. If the pressure at A is eased by 40 KPa, while Jintaining the pressure at B mistant, what will be the new fereuiry deflection? 1 Figure (a) Figure (b) In Figure a, sum-up pressure head from A to B in meters of water: PA Pa 1 0.6 -0.25(13.6) + 0.25 + 2.1 Pe Y = 1.65 m of water CHAPTER TWO G4 _Principies of Hydrostatics In Figure b, py’ = p+ 40 ‘Sum-up pressure head from A’ to B in meters of water; PA (06-3)-(025+ 297136 + (235+) = 2 But MA 165 New mercury deflectio 250+ 2(162) New mercury deflectio Problem 2-41 Im the figure shown, determine the difference in pressure between points and B. serzene CHAPTER TWO. Principles of tyarostatics OD Mercury Sum-up pressure head from A to B in meters of water; FA. + 0.2(0:88) -0.09(13.6) - 031(0.82) + 0.25 -0.1(0.0012) PA. PB = 1.0523 m of water i Y £81(1.0523) = 10.32 kPa biem 2 - 42 (CE Board) suming normal barometric pressure, how deep in the ocean is the point here an air bubble, upon reaching the surface, has six times its volume than {had at the bottom? olution | phe Applying Boyle's Law (assuming isothermal condition) Viz peal p= 1013+ 9.81(0.03)h p= 1013 + 10.108 wav m=1013+0= 1013 va=6v (101.3 + 10.1041) = 101.3 (6 ) 10:104 h = 101.3(6) - 101.3, f= 50.13 m 66 SHAPTERTWo Problem 2 - 43 A vertical tube, 3m long, with one end closed is inserted vertically, with open end down, into a tank of water to such a depth that an open manot connected to the upper end of the tube reads 150 mm of mercury. Neglect vapor pressure and assuming normal conditions, how far is the lower end the tube below the water surface in the tank? Solution ren =A Applying Boyle's Lave Vax paVs Before the tube was inserted; ‘Absolute pressure of air inside, p1 Volume of air inside, Vi = 34 013 When the tube was inserted; Absohite pressure of air inside, Absolute pressure of air inside, p 101.3 +9.81(13.6)(0.15) 121.31 kPa Volume of air inside the tube, V2 = (3- y)A bias aval 1013 GA)=12131[@-y)A] 3~y= 2505 495 m From the manometer shown; Po= Yu lty = 0.81 + 13.6(0.15) ps= 20.0124 kPa. e FLUID MECHANI Principles of Hydrostaties 5 HYDRAUI CHAPTER TWO. Principles of Hydrostatics OF fe the pressure in air insice the tube is uniform ips = pe = 20.0124 kPa wht 20.0124 = 981K; = 04m Then, x=h+y=204+0.495 535 m jam 2-44 ‘consisting of a cylinder 15 em in diameter and 25 cm high, has a neck fs 5 cm diameter and 25 cm long, The bottle is inserted vertically in | with the open end down, such that the neck is completely filled with Find the depth to which the open end is submerged. Assume normal wtric pressure and neglect vapor pressure. n ying Boyle's Law Vin pV the bottle was inserted Volume of air Vi= F057 25) + $6R25) V;= 4,908.74 em! Absolute pressure in air pr= 101.325 Wp the bottle is inserted Volume of air: Va= $15) (25) V2= 4417.9 cm' Pressure in air: pr= 101325 + 9.81) Ins Vs = pe Val 101.325(4,908,74) = (101.325 + 981 hy(4.417 9) 101.325 +9.81 k= 11258 115m r=h+25= 2615 em CHAPTERTWO “HANICS 68 SHAPTERTWo FLUID MECH Mec Principles of Hydrostatics Principles of Hydrostaties &HYOI IRAULICS Peppa 2 48) lementary Problems eer A bicycle tte is inflated at sea level, where the atmospheric pressure ia 1 ee kPaa and the temperature is 21 °C, to 445 KPa, Assuming the lire does expand, what is the gage pressure within the tre on the top of a moun where the altitude is 6,000 m, atmospheric pressure is 47.22 kPaa, and ther report indicates the barometric pressure is 28.54 inches of mercury. Js the atmospheric pressure in pounds per square inch? temperature is 5 °C ane Solution Bh. paVe Bi. Bab ye Jube shown is filled with oil. Determine the pressure heads at B and C in of water At sea level Absolute pressure of air, p= 101.3 + 445 Absolute pressure, p) = = 546,3 kPaa Volume of air, Vi= V Absolute temperature of air. 7) =21 + 273 = 294 °K (On the top of the mountain: Absolute pressure of air; pp= 47.22 + p Since the tire did not expand, volume of air, V: = V Absolute temperature of air, T;=5 + 273 = 278°K Piva. al Th. sete jou s~ 085 5463) (47.22 pw “208 8 ae 4722+ p=51657 ihe tank shown in the figure, compute the pressure at points B, C, D,and E sh. Neglect the unit weight of air Lue =" P Ants: ps = 4.9; po= po = 4.9; pe 21.64 70 Figure 3 - 5: Arch dam ID MECHANICS CHAPTER THREE IRAULICS 83 Total Hydrostatic Force on Surfaces A.A buttress dam consists of a wall, or face, supported by several buttresses on the downstream side, The vast majority of buttress dams are made of concrete that is reinforced with steel. Buttresses are typically spaced across the dam site every 6 to 30 m (20 to 100 f), depending upon the size and design of the dam. Buttress dams are sometimes called hollow dams because the buttresses do not form a solid wall stretching across a river valley. Figure 3-7: Multiple arch dam 4 CHAPTER THREE FLUID MECHANICS. FLUID MECHANICS CHAPTERTHREE gg Total Hydrostatic Force on Surfaces & HYDRAULIGS & HYDRAULICS Total Hydrostatic Force on Surfaces ANALYSIS OF GRAVITY DAM ‘A dam is subjected to hydrostatic forces due to water which is raised on its upstream side, These forces cause the dam to slide horizontally on its foundation and overturn it about its downstream edge or foe. These tendencies are resisted by friction on the base of the dam and gravitatidnal forces which causes a moment opposite to the overturning mment, The dam ‘may also be prevented from sliding by keying its base, A. Vertical force 1. Weight of the dam WiayeVir Wanye Var Wome Vs 2, Weight of water in the upstream side (if any) Wn 1M 3, Weight or permanent structures on the dam 4. Hydrostatic Uplift Us y Ye u Upstream Side Downs Side ne rae) Horizontal Force Hendwater 1. Total Hydrostatic Force acting at the vertical projection, of the submerged portion of the dam, | F=yha Vertical 2. Wind Pressure Projection of — she submerged 3. Wave Action face of dam 4, Floating Bodies 5, Rarthquake Load Il Solve for the Reaction A. Vertical Reaction, Ry Ry= 3h, oes Ry= Wi + Wo + Ws + Wy Un - U: Soni pressure B. Horizontal Reaction, Ry Desrem net IV, Moment about the Toe A. Righting Moment, RM. (rotation fowards the upstream side) RM = War Wim + Wars + Wee Figure 3 - 8: Typical section of a gravity dam showing the possbkfores acting B. Overturning Moment, OM (rotation towards the downstream side) eee OM=Py+ U2) + U2 With reference to Figure 3 - 8, for purposes of illustration, anassumption was made in the shape of the uplift pressure diagram, V. Location of Ry (%) |. Consider 1 unit (1 m) length of dam (perpendicular toe sketch) I. Determine all the forces acting: (CHAPTER THREE FLUID MECHANICS 86 _Totairiydrostatic Force on surfaces HYDRAULICS where: 1 unit weight of water = 9.81 KN/m? (or 10) kg/m!) y= unit weight of concrete w= 24y (usually taken as 23.5 KN/ms) Factors of Safety Factor of safety against sliding, where: ‘oefficient of friction between the base of the dam and the foundation Foundation Pressure For e< B/6 From combined axial and bending stress formula: R, 9B /2 v5 RvenB/2) BB FLUID MECHANICS \PTER THREE CHAI 87 & HYDRAULICS ‘Total Hydrostatic Force on Surfaces R, be y (ial) mieeest/6 Ba Note: Use (+) to get the stress at point where R, is nearest. In the diagram shown above, use (+) to get qr and (-) to get 7 compressive stress and a positive stress indicates tensile stress, A negative stress indicates Gince soil cannot carry any tensile stress, the result of Eq. 3 14 is invalid if the stress is positive, This will happen if e > B/6, Should this happen, Eq. 3 - 15 uill be used, when e> B/6 x aa/y an3x a2 JID MECHANICS CHAPTER THREE gq) eriicen FLUID MECHANI ee HYDRAULICS Total Hydrostatic Force on Surfaces Total Hydrostatic Force on Surfaces & HYDRAULI BUOYANCY = unit weight of the fluid ee Vs me displaced. Volume of the body below the liquid surface ARCHIMEDES’ PRINCIPLE Vp = volume displaced. Volume of the body q ‘A principle discovered by the Greek scientist Archimedes that states that “ body immersed in a fluid is acted upon by an upward force (buoyant force) equal to weight of the displaced fluid”. solve problemts im buoyancy, identify the forces actnng and apply conditions of static qulibriv EFy=0 a EFy=0 is principle, also known ao the Ime of hydrostatics, applies to both float =M=0 and submerged bodies, and to all fluids. homogeneous solid body of volume ¥ “floating” in a homogeneous fluid at Consider the body shown in Figure 3 - 9 immersed in a fluid of unit weight ‘The horizontal components of the force acting on the body are all equilibrium, since the vertical projection of the body in opposite sides is eri same, The upper face of the body is subject to a vertical downward for Vp= Se eLoreedy y. Tet y Eq.3-17 ‘which is equal to the weight of the fluid above it, and the lower face is subj sp-grofliquid fuga to an upward force equal to the weight of real or imaginary liquid above it ‘The net upward force acting on the body is the buoyant force. % If the body of height H has a constant horizontal cross-sectional area such as Wertical cylinders, blocks, ete a i [cross-sectional are, A cs Fo igure 3 - 9: Forces acting on a submerged bocy BE=Fr-Fr =(Vok) -y(Voh) BF = y(Vol:- Voli) If the body is of uniform vertical cross-sectional area A, the area submerged A = ap-profbody 4. Teoly 4 Eq.3-19 sp.grofliquid” Yyqua CHAPTER THREE FLUID cs 90 MECHANI Total Hydrostatic For a ID MECHAN! CHAPTER THREE |ECHANICS 91 i ic Force on Surta ae HYDRAULICS. Total Hydrostatic Force on Surfaces STATICAL STABILITY OF FLOATING BODIES A floating body is acted upon by two equal oppesing forces, These are, df body's weight WV (acting at its center of gravity) and its buoyant force B (acting at the center of buoyancy that is located at the center of gravity of # displaced liquid) When these forces are collinear as shown in Figure 3 - 10 (a), it loats an upright position. However, when the body tilts due to wind or wave actio the center of buoyancy shifts to its new position as shown in Figure 3 - 10 (B and the two forces, which are no longer collinear, produces a couple equal t Wea). The body will not overturn if this couple makes the body rotate towards its original position as shown in Figure 3 - 10 (b), and will overturn if 1 situation is as shown in Figure 3 - 10 () Figure 3 - 10 (c): Unstable postion The point of intersection between the axis of the body and the line of action of Fo o the buoyant force is called the metacenter. The distance from the metacentet (M4) to the center of gravity (G) of the body is called the metacentric hei (MG). It can be seen that a body is stable if M is above G as shown in Figure 10 (b), and unstable if M is below G as shown in Figure 3 - 10 (¢) If M coincides with G, the body is said to be jus! stable Figure 3 - 10: Forces on a lating body RIGHTING MOMENT AND OVERTURNING MOMENT RM or OM= WG) Eq. 3-20 ELEMENTS OF A FLOATING BOD ~ weightof the bod Br = buoyant force alvays equal to Wfor a loatng body) Ge center of gravity of he body Bo center of buoyancy inthe upright position (centri ofthe displaced guid bo" center of buoyancy inthe ted postion Up = volume displaced N= metaenter the pont ofintersaton between the line of aston ofthe buoyent force and te ai fe body <= center of gravity ofthe wexiges (immersion and emerson) 4 Thonznelal distance between the’ ofthe wedges Volume ofthe wedge oF mmersion 8 angle of ting hbo "stance tom Mio Bo Go = distance rom Gto Be MC = metacentric height, distance from M to G Fgure 3 - 10 (a): Upright positon Figure 3 - 10 (b)= Stable postion 92 SHAPTER THE FLUID MECHANIK CHAPTER THREE 93 Total Hydrostatic Force on Surfaces & HYDRAULK HYDRAULICS Total Hydrostatic Force on Surfaces Metacentric height, MG = MB. GB, Moment due to shifting of BF= moment due to shifting of wedge BF (2)=F(s) Use (-) if Gis above Bo BR=y Vo Use (+) iG is below Bo Pete / z= MBo sin® Nore: M as always above 6, f 7 Vo MBa sin 8 = 0s VALUE OF MB, The stability of the body depends on the amount of the righting momes which in turn is dependent on the metacentric height MG. When the body tilt the center of buoyancy shifts to a new position (Bo'). This shifting, also cau the wedge « to shift to a new position v. The moment due to the shifting. the buovant force BF(2) is must equal to moment die to wedge shift F(s) oe Eo For small values of 8, (@ = 0 or Be (72) tan6 ae Wedge, vlume = v Figure 3 - 11: Rectangular body Woatertine Section Consider a body in the shape of a rectangular parallelepiped length 1. as shown in Figure 3-11; Volume of wedge, v Volume of wedge, 0 ¥a(B/2){(B/2) tan OL For small values of 8, = 2B NID MECHANICS CHAPTERTHREE cHaereR tineE eg a ae A HYDRAULICS Total Hydrostatic Force on Surfac Total Hydrostatic Force on Surfaces & HYDRAU! Vp sind HUB ton B<3 B MB, = "3 But for small values of 8, sin @~ tan a Il values of 8, 8 MB. (672)sec0 —_4 (2) tan 8 Note: This formula can be applied to any section. Ccentrald of wedge Since the metacentric height MG is dependent with MB, the stability of From Eq.3 - 22, floating body therefore depends on the moment of inertia of the water rolling bec the moment finer In piching i greater than ‘hat roling me where Ls the nth prendre fare v= VA(B/2)[(B/2) tan OIL v= $LBYan 8 MOMENT ‘The righting or overturning moment on a floating body is: Centroid of triangle, 7 tits _ 04 (B/2)sec8 + (B/2)cos0 From geometry RM or OM=Wx=W (MGsin 8) ~H[teeate 6 (“cos 96 CHAPTER THREE FLUID MECHANIK FLUID MECHANICS CHAPTERTHREE 7 Total Hydrostatic Force on Surfaces LANDRATEL HYDRAULICS Total Hydrostatic Force on Surfaces 2 Consider a pipe of diameter D and (12? tano| 2(2+<02%6 Ihickness be subjected toa net pressure MB, = Buren, p To determine the tangential stress in . (BDE)sine the pipe wall, let us cut a section of length 2 2 falong the diameter, The forces acting on Gane ecase this section are the total pressure F due to Mi eee { the internal pressure and this is to be pee \ tesisted by T which is the total stress of 2 Ls cos? \ the pipe wall. Maes Haast \ Pips ere ) Applying equilibrium condition; 0-2 (ty “ae ca toe F=2r Naga teeta ase ea , F=pA=pDs ae ) it sec? 0 = 1 + tan? 0 Teal 7 (8% 1) B pecan engl pDs= 2x (Sr(6x MB, = <2 are) (2 2 Tangential stress, = 22 12Q)D 1D To determine the longitudinal stress, lot us cut the cylinder across its length as shown, [Fu=0) F=T F=pA STRESS ON THIN-WALLED PRESSURE VESSELS is a ‘THIN-WALLED CYLINDRICAL TANK Ava = xDE A tank or pipe carrying a fluid or gas under a pressure is subjected to tensile T=SinDt forces, which resist bursting, developed across longitudinal and transverse sections SixDt pap D Longitinal stress, $, = 2 Fq.3-27 p= internal pressune~ external pressure cH 98 IAPTER THREE FLUID MECHANICS ‘Total Hydrostatic Force on Su nD MECHANICS CHAPTER THREE lydrostatic Force on Surfaces & HYDRAULICS HYDRAULICS 99 ‘Total Hydrostatic Force on Surfaces ‘SPHERICAL SHELL Ifa spherical tank of diameter D and thick r vckness | contains gas under a pressur of p, the stress at the wall can be expressed as : 7 Problem 3-1 He {vertical rectangular plane of height d and base b s submerged na liquid , Writh its top edge atthe liquid surface. Determine the total force F acting on i { fe side and its location from the liquid surface. Solution ‘SPACING OF HOOPS OF A WOOD STAVE PIPE n(a/2\(bd) Vayb id i aa/2 sake (bd)(d/2) end/6 + Pressure dagram (languiar prism) write yy aa/2+d/6 77 the pressure diagram: Volume of pressure diagram slr) = *ay b 25, Ay Spacing, = 25:40 esa where The location of Fis at the centroid of the pressure diagram, 5,= allowable tensile stress of the hoop x= cross-sectional area of the hoop p= internal pressure in the pipe D = diameter of the pipe For rectangular surface (Inlined or vertical) submerged in a fluid with top edge flushed on the liquid surface, the center of pressure from the bottom is 1/3 ofits height. Note: 100 Fur, O must be below Fu [Mo = 0} Faa(2~y) = Fu~y) G.150/85- 9) = 201144 -y) 1 2n)(6)* anx6y(t3) 0.23077 m Total normal force 106 SHAPTER THREE FLUIDMECHANICS MME F.uio mechanics CHAPTER THREE 4.97 i See ZHVORAULICS MG HVORAULICS Tota Hydrostatic Force an Suraces Solution 17) + 124,800.77) y=363m_ > Location of F from the bottom Fr rha=9sih (015) Fe14715h 00 2Q) = 22m 2m where 7 C5)? (15*1h 12h Pressure Diagram i + 0.08333 = 2.718 5.27 m= h +05=5:77m > critical water depth 3000(8)(2n) = 128,000r kg, a= ¥4,800)(6)(2n) = 28,800x kg P= P,+P=156,800rkg > Total normal force Py=Piy +P, Problem 3 10 (156,800) y = (128,000x)(4) + (28,800n}(2) {A vertical circular gate is submerged in a liquid so that its top edge is flushed with the liquid surface. Find the ratio of the total force acting on the lower .63m > Location of P from the bottom half to that acting on the upper half. Problem 3= aation In the figure shown, stop B will ws. Ratio = A break if the force on it reaches 40 KN. Find the ertical water depth. The length of the gate H Ratio= perpendicular to the sketch is 15m h im aoa OE CHAPTER THREE 108 _Totat Hydrostatic Force on Surfaces FLUID MECHANICS FLUID MECHANICS CHAPTER THREE 4. QQ & HYDRAULICS Total Hydrostatic Force on Surfaces Problem 3 ~ 11 A 30 m long dam retains 9 m of ‘water as shown in the figure, Find the total resultant force acting on the ‘dam and the location of the center of eee pressure from the bottom. det A= 1305 m0? = (9810 x 0.83)(4.167)(1.900) Solution Feyha F=981(45){(30)(10.392)] F=13,763 KN ‘An inclined, circular gate with water on one side is shown in the #4(00)(10.392)° figue. Determine he (Gor 10.392)(45 7 sina te satan e=1732m 8 y= ¥9(10.392) -1.732 . y= 3464 mn y= 4 (10.392) = 3.464 m. lem 3-12 The isosceles triangle gate shown in the figure is hinged at A and weighs 1500 N. What is the total Solution FayhA it =2+05sin6o" hydrostatic force acting on one side 33 of the gate in kiloNewton? F=981(2.433) $ (1? F=18.746 kN ‘ol(s = 0.88) 2m 110 CHAPTER THREE FLUID MECHANICS Total Hydrostatic Force on Surfaces & HYDRAULICS Problem 3 - 14 \ The gate in the figure shown is 1.5 m wide, hinged at point A, and rests against a smooth wall at B. Compute (a) the total force’on the gate due to. seawater, (2) the reaction at B, and (c) the reaction at hinge A. Neglect the weight of the gate Seawater S= 103 Solution PaR+D d=36m tan 0 = 2/3 6=33.09" i sind gebReaes singe 721m (wo Paha F = (9.81 x1.03)(4)(0.5)(3.6)] 18.25 kN FLUID MECHANICS CHAPTER THREE & HYDRAULICS Total Hydrostatic Force on Surfaces 111 Ay (1.5%3.6)(7.21) e=0t5m x=18-035 z=165m EMO) Fé) -RaQ)=0 25(1.65 Ryn 180 kN ee [Fi= 0] Rm+Fsin6-R=0 180 - 218.25 sin 33.69° 58.94 KN Ru a Ryv-Pcos0=0 Ro= 218.25 cos 33.69 Ray= 181.6 KN. Raw Rav? + Ran? = Yt181.6)* + 658.94)" Ry=190.9 KN Problem 3-15 Determine the magnitude find location of the total hydrostatic force acting on the 2m x 4 m gate shown in the figure. CHAPTER THREE Total Hydrostatic Force on Surfaces 112 Solution FapgA P= Sih p Pa. = (281.293) + 81)(15) + @81H080)(1) +32 P= 91.645 KPa s1.645(2% 4) F=73316 KN Solving for ¢ Solve for ft and 7 Feyha 733.16 = (081x126) 24) i =7414m [sins 60° = 7414 / sin 60° 561m HONE! @xa(e5e1) ‘Therefore. P is located 1.844 from the hattam of the gate. (CHAPTER THREE Force on Surfaces 113 ‘& HYDRAULICS Total Hydrostat Problem 3 - 16 (CE November 1997) Determine the magnitude of the force on the inclined gate 15 m by 05 m shown in the Figure 001. The tank of Water is completely closed find the pressure gage at the bottom of the tank reads 90,000 N/m. Use 9800 N/cu. m. for water. Po, = 64030 Pa P= 64030 (0515) F=48,0225 N = 90,000 Pa Problem 3-17 The gate shown in the figure is hinged at A and rests on a smooth floor at B. The gate is 3 m square and oil of having sp. gr. of 0.82 stands to a height of 15 im above the hinge A. The air above the oil surface is under a pressure of 7 kPa above atmosphere. If the gate weighs 5 KN, determine the vertical force F required to open it. UID MECHANICS CHAPTER THREE ‘& HYDRAULICS ‘Total Hydrostatic Force on Surfaces 115 our 114. Eq. (1) Fi=puA Fy = 82{(1.5)(1.5)] = 1845 KN Fa= 1A(7.36)(0.75)(1.5) F=414KN Bayh A F,=9.81h [(.5)(1.5)] Fy=2207h x=075+e 15015)? 2 (aah 1875 i z=o75+ 94875 7 In Equation (1): n07k 75+ 8 )=184.5(0.75) -4.14(1.25) = 0 16.55h1 + 4.138 - 138.375 -5.175=0 1655 ii =842m 39.412 hak -075 h= 767m FLUID MECHANICS CHAPTER THREE & HYDRAULICS 121 Total Hydrostatic Force on Surfaces Problem 3 - 24 Find the magnitude and location of the force exerted by water on one side of the vertical annular disk shown, Solution Feyha 1.814) (1.5)? - x(1)"] 54.1 kN’ Location of F: ow Je 2 4050-50)" Ay A{(15)? - 0)" 14) = 0.203 m ¥p"4+0,203~ 4.203 m below the ws. Problem 3-25 The gate in the figure shown weighs 5 kN for each meter normal to the paper. Its center of gravity is 0.5 m from the left face ‘and 0.6 m above the lower face. Find h for the gate just to come up to the vertical position, CHAPTER THREE 122 Total Hydrostatic Force on Surfaces FLUID MECHANICS & HYDRAULICS Solution { Considering 1 m length Fr=%(9.819(19(0) 905 I? kN = = 281h(15)(1) 14715) kN h [2Mo= 0} Fi(li/3) + W (0.6) - Fx(1.5/2) = 0 4.90516 (1/3) + 5(0.6) -14.715h (0.75) = 0 na =0 Solve hi by trial and error = 0.2748 m Problem 3-26 In Problem 3 - 25, find hr when the force against the “stop” is a maximum. Fi(h/3) +W(06) + PLS) P.0.5/2) =0 4.905h (1/3) + 5(0.6) + P(L5) 14/19 (075) =0 P= 1098-73503 f-sa7p-7288-0 r= 225 he15m ‘Problem 3 - 27 Determine the force due to water acting on one side and its location on the parabolic gato shown using integration, FLUID MECHANICS charreR THREE & HYDRAULICS 123 Total Hydrostatic Force on Surfaces ty Lal ss A= 2x dy bles ame 2 am| cpa Wy /3 aE = yy 22173) ay) aE = 231? dy fe =2a1y forw Fe231y i: ‘| 31(981)2 [3-09] F=141.3kN for ! fibsrwew) Location: Fy wer o3eot fy??24y i ioe yp~ 0604 [2/77 |. Y= 0.1608 2/7) [97-078] 3p~ 2.14 m below the w.s. (CHAPTER THREE Total Hydrostatic Force on Surfaces 124 Problem 3-28 /LUID MECHANICS. HYDRAULICS Probleny 3 - 29 (CE Board) CHAPTER THREE Total Hydrostatic Force on Surfaces 125 In the figure shown, find the width b of the concrete dam necessary to prevent the dam. from sliding. The specific gravity ‘A dam is triangular in cross-section with the upstream face vertical. Water is flushed with the top. The dam is 8 m high and 6 m wide at the base and Weighs 2.4 tons per cubic meter. The coefficient of friction between the base land the foundation is 0.8. Determine (a) the maximum and minimum unit of concrete is 24 and the ressure on the foundation, andl the (0) factors of safety against overturning, p coefficient of friction between the tnd against sliding base of the dam and the fundation is 04, Use 15 as the Solution factor of safety against sliding. Is the dam also safe from BSP. gr: of conc, fan = 2s overturning ? * 24% 1000 Sp. gt of COME, Se = =24 Solution Pe 1000 Consider 1 m length of dam Consider 1 m length of dam Wem te Ve b. W=V We= 12.1060) pestle} Gea la tel We=144 by (v2.4) FE (6)(8)(0) > W=576y — wherey=unit wt of water P=yhA MT ya : K Fe y2251045)0)) ue peyia im fir aaa quae = yx) TN Re= F=10.125 Fer * K RaW. ne N | Heal Toe \ N Fs, Ci “ho is = 24044) RM =230.4y N 10125 OM=P@/3) \ b= 2697 m 2326/3) N s,- SM om Fog Welb/2) = F(15) 14.4(2.637}7(2. TWW.Lz>y(1.5) 2519 = 0.481 m soil pressure at the toe > soil pressure at the heel Solution A. Neglecting hydrostatic uplift: 1. Consider 1 m length of dam face vertical at Wate The specific gravity of 4 m at the top. Hh. Forces Wy = PVs = (1 ¥ 2.4)1(4)(22)(1)) Wy = 211.27 (r= 2491 (B-4)20)(0)] 24By - 96y A= Qoy{20))) 0% We IM, Reaction Ry= 35, = Wit We = 211.2) + 24By - 96y Ry = 24By +1152 CHAPTER THREE Total Hydrostatic Force on Surfaces 128 FLUID MECHANE 2m h-mY vom | sresare| 3 aoram) 1 | | 3 [La | IV Moment about the toe FT ae a RM W(B- 2) + Wa] 2 (B -4)) = 201.248 - 2) + (246-96) [4 (B -4)) 11 2By - 422.4y + 161 - 128By + 256) By + 83.28 y 166-47 (20/3) 2001(20/3) . RM om om V Location of R RL = RM-OM Since the resultant fore wil passthrough the extreme edge of the middle thirds near the toe, ¥ = B/3 Then, a (24By + 115.29)(B/3) = 168% + 83.28) - 166 4y . 1933.33) 8B + 38.4By ~ 160" » 6 2By 1499-73 8B? + 44.88 1499.73 = 0 CHAPTER THREE Total Hydrostatic Force on Surtaces 129 28) B= 11175 m Factors of Safety Factor of safety against sliding: Pee R _ 05)12411.175)1 i 200y FS, = 0.9585 Factor of safety against overturning: RM om 1o(11.175)?y + 83.2(11.175)7 ~ 166.47 3 1383.33 ey FS,=207 A. Considering hydrostatic uplift: Uplift fred U Ry=WitWe-U 2aBy + 115.2y + 10By 14By + 115.2) 1080 20/9(8)(1) Ry RM= WB RM 2+ WA2 (8-4) J6BPy + 88.28 y - 166.4 oM (20/3) + U2B/3) ~ 2007(20/3) + 10By (28/3) (67B% + 1333.33) R, =RM-OM (14By + 115.2/)(B/3) = 168%y + 83.2 4.668"+ 44.88 - 1499.73 =0 yy - 166.4y -(6.67B%y + 1333.334) 24.56) B= 13.766m ER THREE 130. sHarrenmee , pp MECHANICS a corteenest 431 10. Tor Hyarecats Force eta CHOWAN HYDRAULICS Total yao Foundation stress B/S. 13.766/3 = 4.59 m v= B/2~ ¥ = 2.2083 m 81)(6x1) = 17658 kN 4@=2m 14(13.766)(9.81) + 115.2(9.81) 3020.73 kN 76 KN 3.020 ZI, (2.2943) Vs - 2 5[(2)8)(0)] 13766 |'* “13.766 88 KN r= 43887 kPa =O kPa Paes = @/3)Q)= 133m Problem 3-31 (CE Board May 2002) The section of a concrete gravity dam shown in the figure. ‘The depth of water atthe upstream side is 6m. Neglect hydrostatic uplift and use unit weight of concrete equal fo235 kN/m?. Coefficient of friction between the base ofthe ddam and the foundation is 06. Determine the following: (a) factor of safety against sliding, (b) the factor of safety against overturning, and (the overturning moment acting against the dam in kN-m R= F=17658kN RM=Winit Woe 376(3) + 188(1.239) 1378.604 kN-m RM. OM=Fxy = 17658) OM=353:16kN-m > overturning moment 1378.604 _ 5.094 134 Mm 0 ‘CHAPTER THREE FLUID MEcHNics FLUID mecHanics CHAPTER THREE 4 35, Total Hydrostatic Force on Surtaces & HYDRALICS HYDRAULICS Total Hydrostatic Force on Surfaces Consider 1 m length of dam ~ roe BM. 6830032 “OM 377 758 Forces Fsy0181 wan Wi = 2854 (15(6.2)(62)(1)] = 3183 KN (d) Foundation pressure Wo = 23.54 [(7)(52)(1) = 8,569 kN es ae ¢*382/2-132=5.9m 35 (CE Board) ‘The ‘crest gate shown consists of a cylindrical surface of which AB is the base supported by a structural frame hinged at. 0. The length of the gate fs 10m. Corffpate the ‘magnitude and location of the horizontal and vertical components of the total pressure on AB, Fu = Weightascn Ax (4)(2) = 8 me Aa=%en(2)? = 3.14 my A=8+314 A= 1114 m? Solution 865m 3 U4 Z = 84) +3.14(0849) 0.957 m 1060" Therefore; Fy is acting 0.957 to the right of 4 CHAPTER THREE cHarTer THREE FLUID MECHANICS EATERIES. 130) 138 total Hydrostatic Force on Surfaces A HYDRAULICS Total Hydrostatic Force on S Fusyha Fy =9.81(4.39)110(8.66)) Considering 1 meter length: y= 3679 KN Bunya Fy =9.81G)(61) y= 4 (866) = 2.887 m By/= 17658 KN Therefore; Fy is acting 2.887 m above O Fyry¥ c= Ancor - Asia Rey = HOP) 6 sin 60" Vasc ™ Vaoec= Vaow Ayes SI - yy Vaac = 340 (66) x 10-4109 [oor] «10 Ac=326 me CS * Fy=9818.26%1) Fy=9.81(125 90) (= 31.98 KN Fy =1235 kN eB i 5 z Moment about O due to Fy and Fy=0 [a76.58)* + (1.98) Fv) =Fu(y) 79.45 KN 1285 x = 3679(2.877) 2=857m Problem 3-37 Calculate the magnitude of the sesultant pressure on a 1-ft-wide strip of a semicirculaf taintor gate shown Jn Figure-12, ‘Therefore; Fy is acting 8.57 m to from O Problem 336 (CE May 1999) Calculate the magnitude of the resultant force per meter length due to water acting on the radial tainter gate shown in Figure 021 Solution Figure-12 Peg Figure 021 , (62.4 x 25)(5 1) = 780 Ibs 1 Vasc 62.4 (4 @)Q)] = 1225 Tbs Fe VEuP +O P= of(780)* + (1225)* = 1452 Ibs CHAPTER THREE 140 Total Hydrostatic Force on Surfaces Problem 3 - 38 Determine the magnitude of the horizontal and. vertical components of the total force per meter length acting on the three-quarter cylinder gate shown Bin 7 he bh Farha Fx = 9818){(12)] 81[42)(1) + 075{x2"A)L Fy=170.94kN FLUID MECHANICS (CHAPTER THREE & HYDRAULICS Total Hydrostatic Force on Surfaces Problem 3 - 39 (CE Board November 1993) In the figure shown, the 1.20 m diameter cylinder, 1.20 m long is Acted upon by water on the left and ‘ll having sp. gr. of 0.80 on the right. Determine the components of the feaction at B if the cylinder weighs 19.62 KN. Solution Fins yh Fin = 981(1.2)(1.2 x 1.2) Fin = 16.95 kN Fue Fr = 98102 x (0.6%(0.2)1 Fin = 6.657 KN, Fra=1i A= (981 x 08)(06)(1.212) Fin= 678KN Fia= Va (981 x 08)[% x (0.6)°1.2)] Fr =532kN [F:= 0] Fun Fin= Ri=0 Rays = 16.95 - 6.78 Ray = 1017 KN Dv =O] Rev+ Fri + Frn-W=0 Rav = 19.62 ~6.657 -532 Ray = 7.64 KN (CHAPTER THREE 142 FLUID MECHANI FLUID MECHANICS CHAPTER THREE 4 43 Total Hydrostatic Force on Surfaces ‘HYDRAULI & HYDRAULICS Total Hydrostatic Force on Surfaces Problem 3 - 40 81 x 0:82)(0.00628) An inverted conical plug 400 mm diameter and 300 mm long closes a 200 diameter circular hole at the bottom of a tank containing 600 mm of ofl having sp. gr. of 0.82. Determine the total vertical force acting on the plug, ~ o 63.5 N downward ‘2 m diameter horizontal cylinder 2 m long plugs a 1m by 2m rectangular hole at the bottom of a tank, With what force is the cylinder pressed against the bottom of the tank due to the 4-m depth of water? ai 15m Solution LY In =2% (1cps30") m= 173zm =a om In 2.268 m Ayx2 ‘Area, A, = Area of rectangle DEFG ~ Av (1)? (60°) aay) sin 60" ‘Area of segment, Ay= fea Area of segment, Ax= 0.09059. Fi= 81 x 082)fx(0.1)%(0.45)] ‘Area, A= 1(2.268) ~ 0.09059 Fi=0.114kN Area, Ai = 2.1774 m8 aie 21774(2) = 4.355 m? Me 3 F,=9.81(4.355) Va= Vertan = Voy eS, = 2225 (03) +(02)03) (04) 1014038) Vo= 000628 m8 144 SHAPTER THREE Area of segment, Az = Area of segment, Az = 0.614 m? .614(2) = 1.228 m? Fy= Fa=9.81(1.228) Fy=Fy=1205kN Net force = Fy - Fa- Fs Net force = 42.72 ~ 12.05 - 12.05 Net force = 18.62 kN Problem 3-42 In the figure shown, determine the horizontal and vertical components of the total force acting on the cylinder per m of its length, Total Hydrostatic Force on Surfaces & HYDRAULI (1)? (1209) FLUID MECHANICS (CHAPTER THREE & HYDRAULICS, ‘Total Hydrostatic Force on Surfaces 145 nA 81(6.12)((4.24)(0] Fm 254.56 KN ¥4(1)(1) sin 120° 360° Fun Votes Vasicd™ Avett + Amped) 1 Vat ioe eget Vane 40. mi? Fy=981(4041) Fy = 393.38 kN, roblem 3-43 ‘The gate shown is a quarter circle 25m wide, Find the force F just sufficient to prevent fotation about hinge B. Neglect the weight of the gate, FunyhA Fu = 981(1)25 «2) Fy = 49.05 kN Fue Vase a Fy =9.81[(2 2)-025x(2):}@3) Fy= 21.05 kN CHAPTER THREE Total Hydrostatic Force on Surfaces 146 FLUID MECHANICS & HYDRAULICS CHAPTER THREE Total Hydrostatic Force on Surfaces 147 Solve for z and x Since the surface 1s circular, Ma = 0 due to Fy, and F Fi) = Fu 21.05(2) = 49.05(2/3) 155m [eMu = 0] Fi (2/3) + F(x) = £2) =0 2F = 49,05(2/3) + 21,05(0.45) 155 45m P= 21.09 KN Problem 3-44 fe cylindical tank shown has a os hemispherical end cap. 7 Compute the horizontal and = 1™ vertical components of the total --— force dew oiand water acting ‘on the hemisphere a 4 a Solution save = 1293 Forces due to oil Fuo™ Pav Fo = O81 x 0.80)(7 ~ 1.273) » Yan Fo = 6354 KN Fro= Ve z= Volume of imaginary oil above the surface V,= Volume of half cylinder ~ Volume of Ye sphere Vom Yen@)X0) -¥$ 2B) Vo = 70.686 m’ Fvo = (981 x 0.80)(70.686) Fyo = 554.74 KN Forces due to water: Fitw= Poe A Faw = [(981 x 0.8)(7) +9.81(1.273)] « Yin)" Fry = 958.19 KN Fyw= Weight of real and imaginary oil above the surface + weight of real water above the surface Fu = (981 x 0.8)x YinG3)2(7) + 9.81 x A4 x(3)* Fiyy= 1,054.01 Total horizontal force, Fy = Fuo* Frwy Total horizontal force, Fy; = 635.4 + 953.19 ‘Total horizontal force, Fy = 1,588.59 KN > Total vertical force, Fu = Fuw~Fvo Total vertical force, Fy = 1,054.01 - 554.74 Total vertical force, Fy = 499.27 kN ‘Another way to solve for the total vertical force, Fy: Fy = weight of fluid within the hemisphere Fy= Vo Ye Vo Fy= O.81x08)L tx $n G9] + 9.811} $x ())] Fy» 499.27 kN CHAPTER THREE 148 _Totat Hydrostatic Force on surtaces Problem 3-45 Pressurized water fills the tank shown in the figure Compute the ng hydrostatic force acting on the hemispherical surface Solution Convert 100 kPa to its Spee Tend he hye 00 sat fag 10294 m = 10.194-5 h= 5.194 m v, Ye Vols of ylinder + Volume ot hemisphere 194) +h 4x02) Ve = x02} Vy * 82.025 m! F =9181(82025) f= MUR7 KN Weight of imaginary water above the hemispherical surface FLUID MECHANICS CHAPTER THREE & HYDRAULICS 149 Total Hydrostatic Force on Surfaces Problem 3 - 46 Determine the force required to apen the quarter-cylinder gate shown The weight of the gate is 50 kN acting 1.2 m to the right of 0 Hemsphenca surface Since the gate has circular surface, the total water pressure passes Ge reese Ra as, Jocation of the hinge, therefore the noment ‘due to waler pressure About the hinge is zero. nad [Mo = 0} F(2.5) = 50(1.2) + Fx(0) 24kN Lleol 100 Problem 3-47 A hemispherical dome shown is filled with oil (¢ = 0.9) and is attached to the floor by eight diametrically opposed bolts. What force in each bolt is required fo hold the dome down, if the dome weighs 50 kN? CHAPTER THREE 150 FLUID MECHANI Total Hydrostatic Force on Surfaces ‘&HYDRAULI Solution FY Vises she Fy = @81x0.9)[n027 (8) P= 739.66 kN (2) Bho +H Fro = 86.2 KN Problem 3-48 Determine the force F required to hold the cone shown. Neglect the weight of the cone ar 2 ss Lt i FLUID MECHANICS CHAPTER THREE & HYDRAULICS Total Hydrostatic Force on Surfaces Solution fEFv=0) F+ Bae Foi 0 Fe Fa-F; Fo = ¥ Vout ave she com Fou = (9812038)fx(0.805)%5) $x(0.805)@)] 20 [} (1.61)" |= 4072 kN F=63.91-4072 Fe 2319 kN Problem 3-45 ‘4.300 mm diameter steel pipe 12 mm thick carries water under a head of 50m ‘of water. Determine the stress in the steel Solution po ce 981(60)(300) ee) 5-613 MPa [s Problem 3-50 Determine the required thickness of 450 mm diameter steel pipe to carry @ ‘maximum pressure of $500 kPa if the allowable working stress of steo! Solution = Fa 1350) a 124 » 1004 £9.98 mm say 10 mm. CHAPTER THREE 152 FLUID MECHANI lan CHAPTER THREE Total Hydrostatic Force on Surfaces & HYDRAULI Be yrmnticn 153) & HYDRAULICS Total Hydrostatic Force on Surfaces Problem 3 - 51 Determine the stress at the walls of a 200 mm diameter pipe, 10 rm thi lunder a pressure of 150 m of water and submerged to a depth of 20 m in s water Pipe diameter, D = 6 m= 6000 mm Maximum pressure the tank (at bottom), p= Yea p= 9.81(0.8)(7) = 54.936 kPa 2(110%10*)(300) '54.936(6000) 200.23 mm say 200 mm Solution {si= 2) Peto S31q50)"9810.09)20) 12684 kPa = 1.269 MP 1209200) 2110) Problem 3-54 A thin-walled hallow sphere 3.5 m in diameter holds helium gas at 1700 kPa. Determine the minimum wall thickness of the sphere if its allowable stress is 60 MPa. 2.69 MPa Solution Problem 3-52 2D Wall stress, S A 100-mm-ID steel pipe has a 6 mm wall thickness. For an allowable tensil at stress of 80 MPa, what maximum pressure can the pipe withstand? a p pipe citol ee 1000) Solution {28°79 mam Problem 3-55 {A vertical cylindrical tank is 2 meters in diameter and 3 meters high: Its sides fre held in position by means of two steel hoops, one at the top and the other ft the bottom. If the tank is filled with water to a depth of 2.1 m, determine the tensile stress in each hoop. 216) Problem 3-53 Solution ‘A wooden storage vat is 6 m in diameter and is filled with 7 m of oil, s = 0. The wood staves are bound by flat steel bands, 50 mm wide by 6 mm thicl yhose allowable tensile stress is 110 MPa. What is the required spacing oft bands near the bottom of the vat, neglecting any initial stress? Solution 25,Ay Spacing ot hoops, $= pD Allowable tensile stress of hoops, 5,= 110 MPa Cross: ectional area of hoops. Ay ~ 50(6 000 mn CHAPTER THREE 154 Total Hydrostatic Force on surfaces [E Muy = 0} 2733) = F(2.3) T,=03833F > Eg. (1) Peyha F=981 In Eq. (1) 0,3833(43.25) 16.58 KN (tension in the bottom hoop) eV201 = T) = 5.05 KN (tension in the top hoop) Problem 3-56 A vertical cylindrical tank, open at the top. is filled with a liquid. Its sides a held in position by means of two steel hoops, one at the top and the other the bottom, Determine the ratio of the stress in the upper hoop to that in # lower hoop Solution T= F/3 FLUID MECHANIK FLUID MECHANICS & HYDRAULK HYDRAULICS CHAPTER THREE Total Hydrostaicrorce onsurfaces 155 Problem 3-57 A cylindrical container 8 m high and 3 m in diameter is reinforced with two hoops 1 meter from each end, When itis filled with water, what is the tension {in each hoop due to water? Solution | _= [= / if ol Te ha =981(8/2)[8@)] 941.76 KN [2Mup oop $91 21H6)= F (13/3) Te= 135/36 Ta= 13(041.76)/36 Tz = 340.08 KN [EMrottom hop * 0} 27(6) = FG/3) Ty = BE/36 = 5(941.76)/36 T= 130.8kN Tt Problem 3-58 (CE Board November 1982) {A cylindrical tank with its axis vertical is 1 meter in diameter and 6m high. It {s held together by two steel hoops, ane at the top and the other at the bottom. Three liquids A, B, and C having sp. gr. of 1.0, 2.0, and 3.0, respectively fill this tank, cach having a depth of 1.20:m On the surface of A there is atmospheric | pressure. Find the tensile stress in each hoop if each has a cross-sectional area of 1250 mm? CHAPTER THREE 156 _totaittydrostatic Force on surtaces m=0 pee pit vals pee 0+ (y*1)(1.2)=1.2y n= pr Yee Py 1.2y + (7»2)(1 2) = 3.6y P= pat Yai o> 3.6y + (yo3)(1.2) - 7.27 = eqonv.200) B= (1 2911 2)01) = 072) wee Fre phy Fr= 1.2901 2)(1) = Lay Fr=Mgp pyc.2ya) Fie 'a@ov 12n/1 201 = 1 44y Fos pxl.2i0) Fe= 3 6y(1 2)(0) = 4.32) Fs= taps pac aya) Fs = 7 2y - 2670.21) = 2.160 [>My = 0} 3607; (08) + 1.8) = F,(2) + Fy) + Fy8.2) 7.27 = 072y(08) ~ 1 44y(1.8) + 1.44y(2) + 4.32y(3) + 2.167(3.2) Ta= By CHAPTER THREE Total Hydrostatic Force on surfaces 157 stress in bottom hoop FAbt hth ls 27, 0.72) | Lady | Lady + 1.329 + 2167-20867) Ty=144y T; = 1.44(9810) T= 141264 N T4264 Stress, $= t= A, 1250 Stress, $:“11.3MPa > stress in top hoop scien wi 8 icra tps ee Cee eee Problem 3-59 Jin open cylindrical tank of 1.86 m? crostsectional area and 3.05 m high fontsies 2861 liters of water. Into its lowered another smaller tank of the fants eight but of 0.3m? erss section inthe inverted postion, allowing its {pen end t rest op the bottom of the bigger tank. Determine the maximum Meron per vert stinwers on the sides of the bigger tank. Neglot the thickrese ofthe metal forming, the inner tan and assume normal barometric pressure. Solution © Before lowering © After lowering FLUID MECHANI cHarrer THREE 158 & HYDRAULI Total Hydrostatic Force on Surfaces ln Figure © Volume of water = (1.86 ~0.93)(6 +) + 93b= 283 1.86b + 0.98 = 2891 2h+ h=3.044 b=1522-058 > Eq. (1) (hs Viz pe Val 101.325 kPa (atmospheric pressure) 93(3.05) = 2.8365 m? ps= 101.325 + 98th Ve= 0:93.05 - ) 101.325(2.8365) = (101.325 + 9.811)[0.998.05 - 1)] 309,04 = 309.04 - 101.325h + 29,92 -9,81b 29.92h ~ 101,325) - 9.81bh = 0 29.92k ~ 101.925(1.522- 0.5h) - 981(1.522- O.5i)h = 0 29.92h- 154.22 + 50.66 - 14.991 + 4.905h° = 0 4.9052 + 65.65h - 154,22 = 0 = $6653 [EF -AABIENADD 2(4.905) he 2.039 b=05027m Heb+h~2512m The maximum tensile stress occurs at the bottom of the tank 9.81(2542) p= 24.997 kPa = 0.024937 MPa Tension, 7 2= pDA) $D?= 186m? D=1538 m=1/539 mm 27 = 0,024937(1,539)(1) T=192N LUID MECHANICS (CHAPTER THREE 159) & HYDRAULICS Total Hydrostatic Force on Surfaces Problem 3 - 60 (CE Board November 1977) [An iceberg having specific gravity of 0.92 is floating on salt water of sp. gr 1,03. If the volume of ice above the water surface is 1000 cu. m., what is the fotal volume of the ice? Solution Let V total volume of ice volume displaced v-1000 Wee ™ ye V = (9.81*0.92)(V) = 9.0252 V BF = Yaaate Vo BE = (9,81x103)(V - 1000) BE = 10.1083(V- 1000) BAY = 0) Woes BF 9,0252V = 10,1043(V - 1000) 1.0791 = 101043 9364 cu. m. Soamater $= 1.03, Another Solution: For lnuuwgenitous solid body floating on a homogencoue liguict Stodly Yeoay ody yy = Te Fag Tiguia Vs Vieay then; V ~ 1004 006796" V=9,364 cu. m. ‘Problem 3 - 61 (CE Board May 2003, Nov 2002, May 2000, Nov 1992) A block of wood 0,60 m x 0,60 m xf meters in dimension was thrown into the water and floats with 0:18 m projecting above the water surface. The same block vas thrown into a container of a liquid having a specific gravity of 0.90 and it floats with 0.14 m projecting above the surface. Determine the following: (@ the value of ky (@) the specific gravity of the block, and (6) the weight of the block CHAPTER THREE 160 FLUID MECHANI FLUID MECHANICS CHAPTERTHREE 4 6 4 Total Hydrostatic Force on Surfaces & HYDRAULI & HYDRAULICS Total Hydrostatic Force on Surtaces Solution _ Wane In Water: Meet ey Sioa x 0 Draft= eso! tune = 290 ~ 28,204 N/omt er [| foas THe 5 i h-01g= Suen 0.18 > Fa. (1) Problem 3- 63 (CE Board May 1993) A body having a sp. gr. of 0.7 Moats ona liquidl uf sp. gi. 0.8. The volumne of the body above the liquid surface is what percent ofits total volume? wer (3-1) In another liquid Draft= Seed f A Solution Souot 0.98 - 0,126 > Eq, (2) Since the volume of the body displaced (below the liquid surface) is 0. ‘or 87.5% of its total volume, then the volume of the body above the liquid surface is 12.5% of its total volume, [Spoot I= Soe i] i= 0.18 = 0.9h - 0.126 h=054m > height of the block Substitute / to Eq. (1) Syooi(O54) = 0.54 ~ 018, 0,667 > Specific gravity of wood able 3= GF (CE November 1997) Bcc ot woosk 020i tc Ouutig ney watch The bye giaety ot Brood 065 while that of seawaters 168. Find the mintviam aren of Hock Bis seal puvsoctasrattvrtauins ke eos Weight of block = ywoat Vitek Weight of block = (3.81 x 0,667)[(0.6 x 0.6)(0.54)) Weight of block = 1.272 kN Solution Problem 3 - 62 A stone weighs 460 N in air. When submerged in water, it weighs 300 N, Find the volume and specific gravity of the stone, Solution Weight of stone = 460 N Weight of stone in water = 300 N Buoyant force, BF = 460-300 [eFy= 0} BE = Wan + Ws Yas Vent = Wan * Yoo Vora (1000 » 1.02) Vivo = 80-* (1000 » 0.68) Vicon Vivoos = 0.2105 mm? = Area x 0.2 05 square meter {20 ~ ya Vase 160 = 9810 (Van) Vane = 0.0163 Cu. mi CHAPTER THREE 162 FLUID MECHANI FLUID MECHANICS CHAPTER THREE 4 63 Total Hydrostatic Force on Surfaces '& HYDRAULI & HYDRAULICS Total Hydrostatic Force on Surfaces Problem 3 - 65 (CE November 1997) ‘A cube of wood (6.g. = 0.60) has 9-in sides. Compute the magnitude Problem 3 - 67 uniform block of steel (¢ = 7.85) direction of the force required to hold the wood completely submerge will float at a, mercury-water water eee Interface as shown in Figure 27: What is the ratio of the distances ae ee i’ and “b" for this condition? lock Weight of wood = (62.4 x 0.60) (3)° = 15.795 Ibs ‘Buoyant force when completely submerged in water: BF=624(g) = 26325 los Requited force = 23325 - 15,795 Requited force = 1053 Ibs downward Maraag (82 1450) Figure 27 Solution wor FAW Problem 3 66 (CE Nov 2000) aie The block shown in Figure 04 weighs 35,000 Ibs. Find the 4 A value of h, Let A be the horizontal cross-se 2 T e rea ofthe block NominSmtase) 18% Bhi + BF, =W tw Vow + tw Von = 5 V BBI(A x a) + (981 x 1356)(A x b)= (281 *7.85)1A(0* B)) Figure 04 1+ 13.568 = 786+ 7.8 Solution 571 b=6.850 From the figure shown a/b= 0834 (Fr=0) Bis ese Problem 3 - 68 (CE May 1998) 3 o one end ofa 0.1 mx 03 m x 1.20 m wooden Yea Vo ol [fa 5-kg steel plate is attached to one end of a 0.1 le (2AWOs\I2x123) s=08 pole, what is the length of tho pole above water? Use s.g. of wood of 0.50 BF. = 21,5¢5.44 Ibs Neglect buoyant force on stool 21,565.44 + BF; = 35,000 BF:= 1343456 Ibs BFa=%y Vo 13,494.56 = 624 [(12)(12) hi] Ih 1.495 ft FLUID MECHANICS CHAPTER THREE 4 6 & HYDRAULICS Total Hydrostatic Force on Surfaces 164 sHaPrer THREE FLUID MECHANI Total Hydrostatic Force on Surfaces & HYDRAULI ys Solution Neglecting, the buoyant force on steck bs wo = Wea + Wasos TOOO(OA x 03 x y)=5+ 1000(05){0.1 «0.3 x12) y=077m, h=12-y h=12-077 h=043 m Problem 3 - 69 Ifa 5-kg steel plate is attached to one end of a 0.1 mx 0:3 m x 1.20 m woos pole, what is the length of the pole above water? Use sp. gr. of wood of 0: and that of steel 7.85. Solution Word = (1000 + 0.5)(0-1 x 0.3 « 1.2) Weons = 18 Kg Woes #5 kg BF w= 1000(0.1 x03 x a) BFy= 30d BFs= 1000 Vs We = (1000 x 7.85) Vs 0.000637 m? 637 Kg : Vs Woon + Woe ™ BEs + BFiy 1845 = 0.637 + 30d 745 m rh pI Want te Problem 3-70 [A wooden buoy (6.g. = 0.62) is 50 mm by 50 mm by 3 m long is made to float in sea water (6g. = 1.025), How many N of steel (s.g. = 7.85) should be lttached to the bottom to make the buoy float with exactly 450 mm exposed labove the water surface? MBiction asm Wes 2F Wooed ~ Wea = 0 045m BE an + BE vo ef aes BF et = (9810 > 1.025) Vt 8.25 Vie N Yow Vo (6810 x 1.025){(0.05)2.55)] 258 m 64a N Fas i Wc * toot Viet Wreva = (9810 + 0.62)[0.05)°)] Woot = 45.62.N iA Wooet = Yt Vat Wasi = (9810 x7.85) Var Wort = 770085 Vor i a 1005.25 Vani + 64.1 ~ 45,62 - 77008.5 Veai= 0 66953.25 Vow = 18.48, .9810(7.85)(0.00276) Problem 3-71 * [A piece of lead (sp. gr. 11.3) is tied to a 130 ce of cork whose specific gravity is (125. They float just submerged in water. What is the weight of the lead? 166 suarTenninee Sita cr AE sige currence 167 Total Hydrostatic Force on Surfaces & HYDRAULK & HYDRAULICS Total Hydrostatic Force on Surfaces aie Eon ral Tes = BF + BF Hone choses ae 5 grams ai vel ey He Ate, ae BFe= 130 grm Fe Wier Vi nine mote) ce Wotay, iy BFL = yw Vi wy BRL | mG v pe w= 11.3047) (@) Lead is fastened outside the cylinder _(b) Lead is placed inside the cylinder ener es oenve Problem 372 (CE November 1993) osify 20.5) A hallow cylinder 1m in diameter and 2m high weighs 3825 N. (a) Hot BEC = 11.56 kN sun Htc foshen CAV ee una aecdl e ened ie of the cylinder to make it float with 1.5 m submerged in water? (6) How KN of lead if itis placed inside the cylinder? 11.56 + 9.81¥, = 3.825 + 1101 Ve= 0.0772? W, = 110(0.0772) = 8.49 kN (b) Lead is inside the cylinder PFv= 0] Wo Won Br Wi, +3825 = 11.56 W, = 7.735 kN 16g CHAPTER THREE FLUID MECHANIK Total Hydrostatic Force on Surfaces & HYDRAULI Problem 3 - 73, ‘A stone cube 260 mim on each sce and weighing 425 N is lowered into ta Containing a layer of water 1.50 m thick over a layer of mercury. Derm the position ofthe block when it has reached equilibrium. Solution W=425N BE = 1uVon BEy = (9,810 13.6)10.28%()] BF y= 10,459.81 x BEw= Vi BFiy = 9,810(0.28)(0.28 - x)] BE w= 769.1(0.28 ~ x) 028m [sFy= 0} BF BFy=W 10,459.81 x + 769.1(0.28 - x) 9690.71x = 209.652 x= 0.0216 m. 2.6mm ‘Therefore; the block will float with 21.6 mm below the mercury surface, Problem 3-74 A cube 2.2 feet on an ‘edge has its lower half of 59. = 0.8 = 1.6 and upper half of'sg.= 07, Itrests ina two-layer fluid, with lower sg = 14 and upper sg. 08. Determine the height 1 of the top of the cube above the interface. See Figure 33, "22 Figure 33, [LUID MECHANICS CHAPTERTHREE 4 6g HYDRAULICS. Total Hydrostatic Force on Surfaces k—— 2.28 ——I = (624 x 1.4)(2.2)%22 - i] + (624 « 0.8){2.27°0)) (08 = 1.4h + 0.8) (624% 1.122°(1.2)] + (624 ) soliton PyRA~9610)201) . + o0762—0.1504 3. 25 = 0.0962 F— fry: 5.232 kN Fam the FBD of the terete block: Figure (o) We have to assume that the boat have a constant cross-sectional area A below the water surface and use Yyatx = 1000 kg/m* In Figure (0) Wewse = 9.81 Vee BR=W W= Yee Vion = 23.6 Veen Ym Vor 5.232 +981 Ven = 23.6 Vee W= (1000 «1.03)/A0)} Veone = 0.3796 mi w= 10304D 174 SHAPTER THREE FLUID MECHANI \UID MECHANICS CHAPTER THREE Total Hydrostatic Force on Surfaces & HYDRAU HYDRAULICS Total Hydrostatic Force on Surtaces In Figure Bion Bae For any floating body; Buoyant force ttm Vo W = 1000[4(0+0.0762)} Solving for displacement in sea water w= 1000A(D + 0.0762) > Fa.) Youu Vin = W In Figure (¢ Bhs=W- 72730 1000[A(D -0.0762)| = W'-72730 > Eq. @) Solving for displacement in fresh water Yin nay Yoo = W From Eq, (1) and Eq, 2 (62.2)(Vox) = 24,000 x 2240 we Gat neate 1030AD = 1000A(D + 0.0762} Four (2) Ree Let Ibe the difference inthe drafts in fresh & seawater D= 254m (draft in sea water) Valse sintiaa 864,308.68 ~ 840,000 From Eq, (1) a eee b= 076ft W= 2616.24 Draftin fresh water, D=34 +076 ~ 3476 ft From Eq, @) 10004 (2.54 - 0.0762) = 2616.24 - 72730 See eee ‘Problem 3 - 80,(CE Board November 1995) ere A=a7723 nm! Consider an arbitrary shaped body with a submerged volume Vs and 2 density p., submerged in a fluid of density py What is the net vertical force on the body due to hydrostatic forces? W= 1,288,529 kg (9:81/1000) W= 12288 KN ton Fat = Vs ee pxe Problem 3-79 A ship having a displacement of 24,000 tons and a draft of 34 feet in ocet centers a harbor of fresh water. If the horizontal section of the ship at waterline is 32,000 sq. ft, what depth of fresh water is required to float Problem ship? Assume that marine ton is 2,240 Ib and that sea water and fresh wat weight 64 pef and 622 pef, respectively Fa = py Vs {\ spherical balloon, 9 m in diameter is filled with helium gas pressurized to 111 KPa at a temperature of 20°C, and anchored by a rope to the ground. Neglecting the dead weight of the balloon, determine the tension in the rope Use R = 212 m/°K for helium gas and Yyy = 11.76 N/m? 176 —crAbreR Tree Total Hydrostatic Force on Surfaces & HYDRAUI Solution 11x10 RT 787 N/m? 4149/29 3817 m' BE = 11,76@81.7) W= tran Vi W=1,787 81.7) = 682.1 N 4488.8 - 682.1 = 7 T= 3806.7 N 88.8 N Problem 3-82 The buoy in Figure 3 - 1 has 80 N of steel weight attached, The buoy hi lodged against a rock 2m deep. Compute the angle @ with the horizontal which the buoy will lean, assuming the rock exerts no moment on the buoy. Solution Uae TS ae | HYDRAULICS LUID MECHANICS CHAPTER THREE Total Hydrostatic Force on Surfaces We =45.62N (from Figure3- 1) BE= Yo Yo BE = 9810(1.025)(0.05)L] BP = 25.1381 [=Mp = 0} W(.5 cos 0) - BF[(L/2) cos 8] = 0 45.62(1.5) - 25.138L(L/2) = 0 1257 P= 68.43 L=223m sind =2/L 4 sin = 2/233 0-59" Problem 3-83 A right circular cone is 100 min in diameter and 200 mm high an Nin air, How much force is required to push the cone (vertex downward) Jinto a body of liquid having sp. gr. of 08, so that its base is exactly at the Surface? How much additional force is required to push the base 10 mm weighs 16 below the surface? Solution The required downward vertical force is Br-W BE = Yuga Vee BE = (810 « 8) [(n BE=411N 0.1/2540.2)1 F=41-16 Fe 251N Note: This force F= 251 N becomes onstant no matter how deep further the cone is submerged 17g SHAPTERTHREE =“ FLUID MECHA FLUID MECHANICS CHAPTERTHREE 4.79 Total Hydrostatic Fofce on Surfaces &HYDRAI HYDRAULICS Total Hydrostatic Force on Surfaces Problem 3-84 / Solution To what depth will a 2-m{tiameter log, 4m long and of sp. gr. 0.425 si fresh water { Solution For a homogeneous solid body floating on a homogeneous liquid: V2 Vins Fuqua ‘As= 0.425n72 (shaded area) Let V = volume of wood From geometry tn water As Ase Bre =4) Oaasar = v8 Bi\-W-F=0 O,-sin 0 = 267 9siov-W = 40 10eW > Ba Solve 4 by trial and error: 9810 a Try0= 170" i 170" (/180")-sin170° = 276 (2.60) ene Try0= 16 Bry-W-F=0 166"(n/1807)-ninN66%=2655. (62467) (810 «1.3)V-W=100 Try 0= 166.44° 166.44°(x/180°) -sin166.44°=2.67 OK her-y 1- (0) cas (8/2) 1- (1) cos (166.44°/2) From Eq, (1): 0.882 m, y= 40160 9810 v= 0.0204 m* Problem 3- 85 AA block of wood requires a force of 40 N to keep it immersed in water and force of 100 N to keep it immersed in glycerin (sp. gr. ~ 1.3). Find the weight and sp. gr. of the wood, W __160 Unit weight ¥= > =T 750g 7843 N/m? 7843 Unit weight, y Sp. gt Sp. er. (CHAPTER THREE 180 Problem 3 - 86 A recngalat EA peti iced 5 m, as shown, contains oil of sp. gr 08 and water. (a) Find the depth of oll, hk. (B) If a 1000-N block of Wood is floated in the oil, what is the rise in free surface of the water in contact with air? Solution Figure (a) (a) Depth of oil: (Refer to Figure a) ‘Sum-up pressure head from oil surface @ to water surface @ in m of wat Pa Pus nossa 1 0+08h-1=0 91.25 m Total Hydrostatic Forcé on Surfaces Figure (b) (b) Rise of the water surface: (Refer to Figure b) BFW Yea Vo (9810 x 0.8) Vis= 1000 Vo=01274 mm (CHAPTER THREE Total Hydrostatic Force on Surfaces 181 ce the volume of oil remain unchanged Vox = Vota (0.5)@)(1.25) = (05)(5)0") 0.1274 i= 1.301 m {As shown in Figure b, if the oil-water interface drops by a distance of y, the free surface of water will rise by y/2, since the cross-sectional area of the right compartment is twice that of the left compartment Suin-up pressure head from ol surface to water surface in m of water 0+1301(08)+ @-y)-4-4/2=0 1.0408 -1-3y/2= 0 ay/2= 0.0408 y/2= 00136 mor13.6 mm ‘Therefore; the free surface of water will rise 13.6 mm. | tank 350 mm in diameter and 1.8 m high is inserted ically into a body of water with the open end down and floats with a 1300 block of concrete (sp. gr. = 2.4) suspended at its lower end, Neglecting the ight of the cylinder, to what depth will the open end be submerged in BFewe+ BFo-W=0 > Eq. (1) BP es Ver aor # Vion = Weone BF Yeone 1.6m Vem | |_|} y 8102.4) on Foy Veoue = 0.0552 m# BF oe = 9810(0.0552) BF co = 541.7 N BEgt™ Youu Vo BFyi = 9810[$ (0.95)H) BE.) =943.83 h CHAPTER THREE 182 _Totartiyarostatic Force on Surfaces From Eq, (1) ‘SAL.7 + 943.831, = 0,808 m Applying Boyles Law (taking psim = 101.325 kPa) Before insertion: ‘Absolute pressure in air, p= 101.925 kPa Volume of air inside the cylinder, Vi = ¥ (0.35}(0.18) Volume of air inside the cylinder, Vi = 0.0173 m* After insertion: Absolute pressure in air, p= 101.325 + yh Absolute pressure in air, p= 101.325 + 9.81(0.803) Absolute pressure in air, p= 109.2 kPa Volume of air inside the cylinder, Vs $0357 x 0.0962" Volume of air inside the cylinder, V2 po Va] 25 0.173) = 109.2(0.0962x) x= 167m xhty= 18 y = 18-167 + 0.803 y= 0984 m ‘Therefore, the open end is submerged 0.933 m below the water surfa BF, + BF-Wi-Wi=0 Problem 3-88 (CE Board) BE,= Yo Vo A cylindrical buoy 600 mm in diameter and 1.8 m high weighs 205 kg. I = (1000 «1.03){§ (0.6)(0:96)] moored in salt water to a 12 m length of chain weighing 12 kg per m of BE, = 27958 kg length, At high tide, the height of buoy, protruding above water surfa poe 0.84. What could be the length of protrusion of the buoy if the tide dropy 2 Yow Vetain 2.1m? Density of steel is 7,790 kg/m. Use density of water = 1000 kg/m! ane eee 1a CHAPTER THREE Total Hydrostatic Force on Surfaces UID MECHANICS HYDRAULICS 183 Figure a: High Tie Foe b Low Te Weight of chain = 12 kg/m Density of steel = 7,790 kg/m? Volume of steel (chain) = 12/7790 Volume of stee (chain) = 0.00154 Jn Figure a BA 1 We 121 279.58 + 1,586L,- 205-121 = 716m Depth of water, H= 1. +096 Depth of water, H= 812m CHAPTER THREE CHAPTER THREE 184 FLUID MECHANI 185 Total Hydrostatic Force on Surfaces & HYDRAUI Total Hydrostatic Force on Surfaces In Figure lution Depth of water, H’ =H-21 Depth of water, H’ = 6,02 m Weight of bal: Draft, D = H’-1/ W= Yoon Vout Draft, D= 6.02- 1 W= (9810 x 0.42)4 x(0.15° EFv-0) pieeren BP) + BP2-Wh-W: Buoyant Force: BE» (1000 « 1.08) )LF (0.6? BE = Yet Va BE’, = 291.23 (6.02-L) BE = (9810) $x(0.15P BP’ =1753:18- 291.251" BP = 138.69 N ‘BE = (1000 x 1.08)[0.00154(L")] Depth of poot BP) = 1.5861 Work done by W= Work done by BE W.= 121 1753.18- 291,231 + 1.586L’ -205-121"=0 U=513m D=602-5:9 Yanan ‘A hydrometer weighs 0.0214 N and has a stem at the upper end which is 2.78 yn 18-089 um in diameter. How much deeper will it float in oil (sp. gr = 0.78) that in Y=O51m. (length of protrusion) leohol (sp. gr. =)).821)? lution Problem 3-89 (CE Board) PF isoubt ‘A wooden spherical ball with specific gravity of 0.42 and a diameter of mm is dropped from a height of 4.3 m above the surface of water in a pool unknown depth. The ball barely touched the bottom of the pool before: 657 «106m? began to float. Determine the depth of the pool (9810 x 0.78)Voy= 0.0214 Vee = 2.797 x 10° me? Neshol ¢=0.821 Ol, = 078 186 CHAPTER THREE FLUID MECHANI CHAPTERTHREE 4 37 Total Hydrostatic Force on Surfaces & HYDRAULI Total Hydrostatic Force on Surfaces Problem 3 ,93~/ ‘i ‘ A plate oe dene 1 yell op. ORE AED vetcaly in vine. Bice of wood op. gr: = 6 isin the shape ofa retangult pardlllepiped ccube stable? Solution Having a 10-cm square base. Ifthe block floats in salt water with its square fuse horizontal, what is its maximum height for stable equilibrium in the The body is stable if Mis above G. Draft, D= 221, Draft, D= 0.821 4 LL? upright position? Solution Note: The body is stable when M is, above Gand unstable if Mis below G. With smaller value of H, the me,» Yo ‘MB, = 0.102 L Cx ty(0820) GB, GB, L/2-D/2 0.091. Since MB, > GB., Mis above G. The body is stable. Problem 3 - 92 ‘A solid wood eylinder of specific gravity 0.6 is 600 mu mm high. Ifplaced vertically in oil (sp. gr. = 0.85), would it be stable? Solution sp.grwood |, sp.gr.oil Draft, D = 242 (1200) = 847 mm Draft, D = | L MB, | Yo € ol $(200)* 3 Rie tc is Sel Teel “(3007 (847) aioe al MB, = 26.56 mm 3 op GB, = 600 - ¥s(847) GB, = 1765 ince MB, < GB, the metacenter 1s below Therefore, the body is unstable. Inetacenter M will become higher ithan G making it much stable When H increases, M will move Mown closer to G making it less lable. Hence, the maximum. height for stable equilibrium is When M coincides with G, or MB. = GB, Waterline Section From the figure: GB)= H/2-D/2 Draft, D = S84 H = 0.6211 R= OSH OBPIH/? 0.189H wm in diameter and 1 «= 200 nm 5(10)(10)* ** “(10)(10)D 100 _ 13419 es "Toei "A +m (0.62) Harter THREE FLUID MECH (CHAPTER THREE 188 Au Total Hydrostatic Force on Surfaces 189 Total Hydrostatic Force on Surfaces & HYDRAULI [s, = 6B, Initial motacentric height, MG = MB,~ GB, 13419 ; Initial metacentric height, MG = 77.49 - 117.45, Hee Initial metacentric height, MG = -39.96 mm H=843.em joblem 3-95 Problem 3-94 rectangular scow 9 m wide, 15 m long, and 3.6 m high has a draft in sea ‘A wood cone, 700 mm diameter and 1,000 mm high floats in water with ater of 24 m. Its center of gravity is 2.7 m above the bottom of the seow. vertex down. If the specific gravity of the wood is 0.60, would it be stabl Determine the following (@) The initial metacentric height, (0) The righting or overturning moment when the scow tilts until one side is just at the point of submergence. Determine also its initial metacentric height. + x(850)2(1000) Vieont * 128,281,700 m? = ) Initial metacentri height Yo ee Bf tan? Blt. Yo=06 128281700 g| | where 0 = 0° Vo=76569,020 mm Ff, tan? MB, = 2.8125 m wnt ( 1000 ) Va TD )e Vent ee CB.-27 12°15m Initial metacentric height, MG = MB. GB, Initial metacentric height, MG = 2.8125~15 Initial metacentric height, MG () Waterline Section tan = 43 o= 1498" Be mets 49mm MB. = Bh ‘) Vo 76,565,020 wl 2 From the Figure # [27457 GB, = 750 - 3/4 p= paa| cae GB, = 750 - 3(843.4)/4 CB, ~ 117.48 mn Str acie Since MB, < GBy Mis below G and the cone is UNSTABLE. 190. SHAPTER THEE FLUID Mec! FLUID MECHANICS CHAPTERTHREE 4.94 Total Hydrostatic Farce on Surfaces SHYDRAL & HYDRAULICS Total Hydrostatic Force on Surfaces Metacentric height, MG = MB, - GB, Along longitudinal axis (rolling): Metacentric height, MG = 291-15, Since MG> MB,, the moments righting moment 4 W=BF=Wo W= (981 + 1.03)9(15)2.4)] = 3.2738 kN Righting moment, RM = 3278.41) sin 14.93") Righting moment, RM = 1.1893 kN-m 545 m Metacentric height, MG = 5.45 ~ 3.235 Metacentric height, MG ~ 2.215 m_ (the barge is stable in rolling) Problem 3-36 ‘Along transverse direction (itching) A barge floating an fresh water has the form of a parallelepiped havi gee z dimensions 10 m x 30 m by 3m. It weighs 4,500 kN when loaded with cet B,= 2—|1+!%2| where o = 0° of gravity along its vertical axis 4 m from the bottom. Find the metacent me Dl ee height about is longest and shortest centerline, and determine whether of is Riatarae abe fa SOE oy ago Ta053) Solution Metacenrc height, MG = 4902-9235 A | 4 Motacenti height, MG ~ 45.785 m (the barge ie stable in pitching) oO Problem 3-97 (CE August 1973) na ‘crane barge, 2an long, 8 meters wide, and 2 meters high loaded atts center my this road llr weighing 20 short to, oats on fesh water with a draft of Too shen and has fa center of gravity located along its vertical axis ata eae point 1.50 meters above its bottom. Compute the horizontal distance out to | at ks i pile from the centerline ofthe barge through which the crane could swing a the 20s ld shih i hed lifted from the center ofthe deck, and tip the Sold forthe dah, 0 urge with he 2-meter edge jut touching the water surface? ier Von W 5.1 10 «30» | = 4500 D153 GB=4-D2 Ga = 4153/2 GB, = 3.235 m CHAPTER THREE 192 _Totairiyarostatic Force on surfaces Solution ke iE ic a 00 981 TI We 176.58kN, [rene Road oer Wy = 20 shor one BE= yo BF=981[8 x 1.2 « 20) BF = 1,883.52 kN =W; Weight of barge, We = BF ~ Ws Weight of barge, We = 1,883.52 - 176.58 Weight of barge, Wy = 1,706.94 kN Tilted position tang = aR e=na0 Solve for the new position of G in the tilted position: vad) W405) 1.889 52(05) d=0552m CHAPTER THREE Total Hydrostatic Force on Surfaces 193 ~ 12.2) 2 4.533 — 0.848 = 3.685 m 2 [sets ‘MG sin ® 3.685 sin 11.31° = 0.723 m [EMc = 0] (BE) x= Wa(L +2) 1,883.52(0.723) = (176.58)(L + 0.108) L=7.604m > Horizontal distance from the center of the deck jem 3-98 wooden barge of reciangular cross-section is 8 m wide, 4 m high, and 16m ong, is transporting ip seawater (5 = 1.03) a total load of 1,500 kN including its own ght und cargo” Ifa weight of 75 KN (chided in the 1,500-kN) is shifted + stance of 25 m to one side, it will eause the barge to go down 450 mn in the ig of immersion ad also rise 450 man in the corresponding weige of emerson The barge floats vertically (on an even Kee!) before the shifting of the weight Gompute how far above the waterline is the center of gravity ofthe loaded barge ution po! W = 1500 kv 194 CHAPTER THREE FLUID MECI UID MECHANICS CHAPTERTHREE 95 Total Hydrostatic Force of Surfaces &HYDRAL HYDRAULICS Total Hydrostatic Force on Surfaces Solve for the draft, D: alah ye waterline section of a 1,500-KN barge is as shown. Its center of gravity is pats Lys 16+ ree os cieaieanelad anes inst rolling, In the Tilted position 0-6. ae T= Irectangle * Fuiangle * Hemveircte = £128) + OA « 2+ FA a= Pf tant = 6768 mate | [BF=" i 463m 82, , tan? az" 1200.16) 2 1A25(E) + 75(0) = BFC) (B, sin 0 4.63 sin 6.42" 0518 m IMG = MB, - GB,) MG = 4425-15 MG= 2925 m > initial metacentric height A2 sin 642° +25 cos 642° LA25{(h + U.58) sin 6.42%] + 75(2:867) = 1,500(0.518) j= 2987 m > distance of G from the ws 196 CHAPTER THREE FLUID MECH, Total Hydrostatic Force 6n Surfaces & HYDRAULE ——= 7 Supplementary Problems Problem 3 - 100 A vertical rectangular gate 2 m wide and 1.2 m high has water on one with surface 3 m above its top. Determine the magnitude of the hydrostatic force acting on the gate and its distance from the water surface Ans: F = 84.0 KN, yp = 3.63) UID MECHANICS CHAPTER THREE 4.97 HYDRAULICS. ‘Total Hydrostatic Force on Surfaces blem 3 - 104 ater in a tank is pressurized to 80 cmHg. Determine the total force per meter klth on panel AB. ‘Ans: 482 KN Problem 3-101 A vertical semi-circular area of radius r is submerged in a liquid with diameter in the liquid surface. How far is the center of pressure from liquid surface? Ans: 0. blem 3 - 105, the figure shown, the 8-ft-diameter cylinder, 3 feet long weighs 550 Ibs and xfs on the bottom of a tank that is 3 feet Iong. Water and oil are poured into Whe left-and right-hand of the tank to depths 2 feet and 4 feet, respectively. stermine, the magnitudes of the horizontal and vertical components of the ce that will keep the cylinder touching the tank at A. Problem 3 - 102 An open vat holding oil (s = 0,80) is 8 m long and 4 m deep and has trapezoidal cross-section'3 m wide at the bottom and 5 m wide at the Determine the following: (#) the weight of oil, (b) the force on the bottom of vat, andi (c) the force on the trapezoidal end panel. ‘Ans: (a) 1002 KN; (8) 752. (230 Ans: Fy = 749 b> ’ Fy = 2,134 Ibs 4 Problem 3 - 103 Freshly poured concrete approximates a fluid with 20 sp: gr. of 240. The figure shown a wall poured between wooden forms which are connected by six bolts. Neglecting end effects, compute the force in the lower bolts al Ans: 19,170 tbs a Problem 3 - 106 "Compute the hydrostatic force and its location on semi-cylindrical indentation JCD shown, Consider only 1 meter Iength of cylinder perpendicular to the figure below. Ans: Fy = 109.5 KN @ 1.349 m below D #y= 20 KN @ USI mo the left of b 200 H/2): y2 (Some uid soiled) ve=o Lg Stace jst teaching the to im (Wo aud eile) Vortex at he bttom (Some leg sped) yon Vortax (magina) below the bosom (Gore sie) ID MECHANICS HYDRAULICS CHAPTER FOUR rium otuiguids 2OT For closed cylindrical containers more than halffulh of. tiquid, rotated about its vertical axis (I> H/2) ya>0 (ah maginary parabola above) Note: For closed vessels here can ever be any liquid spilled, so the initial Volume of guid (blore rotation) is always equal to the final volume ofthe Iiqud (after rotation) oth inital zone fi inside ts alo te ial volun ofr insite. The vole of ir elton is more ‘convenient to use in solving this type of problem, i= (D/H - Ky K= In /2D (liu uae st teuchina het in) y=H0 (erent ein he eo) i] y> tao (Worex bal ‘the Betton) CHAPTER FOUR 208 FLUID MECHANIK ‘CHAPTER FOUR Relative Equiliprium of Liquide & HYDRAUL Feeney, 209 HYDRAULICS Relative Equilibrium of Liquids tube revolved about ts own axis Note: the pressure head at any point in the tube isthe versal distance from the ‘ube tothe paraboloid. The pressure is. postive if the paraboloid is above the Point and negative if itis below the point. The limiting pressure is absolute Zero. without preSsue at top ‘with preskure at top For pipes and tubes: \Without intial pressure inside it intial pressure inside 210 HAPTER Foun FLUID MECI Relative Equilibrium of Liquids ‘& HYDRAUI [Solved Problems Problems 1 An opensstangular tank mounted on a truck is 5 m long, 2 m wide and 2, high is fitd with water to a depth of 2 m_ (a) What maximum horizs acceleratin can be imposed on the tank without spilling any water and. cletermin the accelerating force on the liquid mass? (c) If the acceleratio Increased 6 m/s’, how much water is spilied out? Solution a) The figre shows the water level under maximum a when no water i spillecout tant= 98 = 02 8 1 =12(9.81) 11962 mvs" 0 Accelerating Force, F= Ma Mass, M = p(Volume of liquid) Mass, M = 1000(5 x 2» 2} Mass, M = 20,000 kg Accelerating Force, F = 20,000 « 1.962 Accelerating Force, F = 39,280 N f= 981(23 1250) -981(22 1150)] = 39.20 KN ID MECHANICS CHAPTERFOUR 4 4 HYDRAULICS. Relative Equilibrium of Liquids (0) When a= 6 m/s 6 981 s tnd 5 coB1.45° = 40875 < 5m Via = ¥44.0875)(25)2) 0.22. m? {A closed horizontal cylindrical tank 1.5 m in diameter and 4 m long is fompletely filled with gasoline (sp. gr. = 0.82) and accelerated horizontally at3 n/s*. Find the total force acting at the rear wall and at the front wall of the lank. Find also the accelerating force on the fluid mass, Solution tano= = + 3 381 123m 1.223 +075 1973 m, Fear 21 A Fre = (9.81 x 0.82)(1.973)[ 5 (1 Fas = 28.05 KN Feo #71 A Fi = (981 x 0:82)(0.75) [4 (1.5)41 212 Harter Four ¥ Relative Equilibrium of Liquids & HYDRAUI Accelerating Force F=Ma Mass. M = o(Volume) . F = 4(1O! « 082){ § (1 5914) 13 += 17390 } 17.39 kN F = 2805 - 10.60 F ©1736 kN Problem 4-3 Seer a A closed rectangular tank 4m long, 2:m wide, and 2 m high is filled wi water to a depth of 18m If the allowable force at the rear wall of the tank 200 KN. how fast can st be accelerated horizontally? Solution Fayha 200 = 9.81 fi [2(2)} 1m 1m By similar triangles 4a, 4-w=4h > Fay CHAPTER FOUR Relative Equilibrium of iquias 213 Also: Vos crgnay ™ Vase ia 4(0.2)@2) = (1/2)x22) x= 16 > Eq. (2) 2=16/x > Eq. @) Substitute z and xz to Eq. (1) 4(16/x)-16=41x > multiply by x 64-16. — 412 Aly +16x-64"0 ‘open tank 1,2 m square weighs 3425 N and contains 0.91m of water. It is id by an unbalanced force of 10400 N parallel (o a pait of sides. What is force acting in the side with the smallest depth? Solve for aand yf F=Ma= 10400 M= Mess + Mant ‘M= 1,000)(1.82)(1.82)(0.91)} +3425/9.81 M=3,36342 kg 10,400 = 3,363.42 x a a= 3.092 m/s CHAPTER FOUR 214 IID MECHANICS CHAPTERFOUR "9 4 5 Relative Equilibrium of Liquids HYDRAULICS Relative Equilibrium of Liquids Aqne = 5.11 me Vane =5.11(1.5) = 7.665 m? 3092 | _y 981 O91 =0.29m<0.91 m OK) oly 62m Vien = 12.4635 - 7.665 Vig = 4.7985 m? Vics = 8.6985 - 4.7985 Vigna ~ 3.9 me P= 9,810 (0.62/2) [0.62 x 1.82) P=3432N Problem 4-5 blem 4 - 6 (CE Board) vessel 3 m in diameter containing 24 m of water is being raised. (a) Find the essure at the bottom of the vessel in kPa when the velocity is constant, and ) find the pressure at the bottom of the vessel when it is accelerating 0.6 m/s? ‘An open trapezoidal tank having a bottom width of 3 m is 2 m high, 1 wide, and has is sides inclined 60° with the horizontal Ii filled with toa depth of 15 m. If the tank is accolerated horizontally along its len 4.5 m/s?, how much water is spilled out? Solution For vetical motion path (lta) he=2am 45. tno=4 = (@) When the yelocity is constant, a = 0, then = 24.64" perk 812.4) 544 kPa (pressure at the bottom) Vapites = Vorg~ Vise a Vora = 2428 (15) x15 (b) When «= 0.6 m/s? (use “+” for upward motion) 8.6985 m? sical (t+ ae p= 24984 kPa = Vaoco= Vane Vasco = 533998 (2)(1.5) Vanco = 12.4635 ms! Vane = Aane(15) ‘ane = #5 (AB)(AB)sin © a= 180° - 60° = 24.64" = 95:36" AE= sin 60° AE= sin60° = 4.618 m Problem 4 = [A vessel containing oil is accelerated on a plane inclined 15° with the horizontal at 12 m/s? Determine the inclination of the oil surface when the ‘motion is (a) upwards, and (b) downwards. Ange = ¥4(5.309)(4.618)sin 24.64° clarrer Four FLUID MECHA cHarre! 216 i jie LT Relative Equilibrium of Liquids HYDRA Relative Equilibrium of Liquids Solution ( Downward motion with a positive acceleration (use “-" with a=-+8 m/s!) tano = —tH p= (981 x0.8)(3) (1-8) stay san 34 kPa ay = COS a ‘ay = 1.2.cos 15° {d) Downward motion with a negative acceleration (use “-" with a= -8m/s") fy = 1.159 m/s? p= 081% 08)Q) (1-sf.) ay asina pnd2.74 kPa v= 12sin 15° av = 0.31 m/s? (a) When the motion is upwards: A cylindrical water tank used in lifting water to the top of a tower is 15 m with a positive acceleration, and (a) downward with a negative aceleration fag 1.159) high. If the pressure at the bottom of the tank is must not exceed 16 KPa, what 3814 (6051) jyaximum vertical acceleration can be imposed in the cylinder when itis filed 0 6539" ith water (b) When the motion is downwards: Solution RFRA prvk(+a/s) 981 (031) 16" 981(1.5)(1 + 4/981) 8 = 6.955" 857 m/s? Problem a-8 lem @-i0 {An open tank containing oil (sp. gr. = 08) is accelerated vertically at 8m ‘An open cylindrical vessel having a height equal to its diameters halfille with Determine the pressure 3 m below the surface ifthe motion is (a) upward [ler and revolved about its own vertical axs with a constant angular spoed of 4 positive acceleration, (#) upward with a negative acceleration, () downw 120 rpm. Find its minimum diameter so that here can be no liquid spilled Solution Solution Bo that there's no liquid spilled, base ofthe : paraboloid rust just coincide with the The pressure at adepth his given by, p =i(: 1 4) pper vim ofthe cylinder. Since the cylinder 8 Is initially half-full, the height of the (@) Upward motion witha postive acceleration (use “+” with a= +8 mj Paraboloid is therefore equal tothe height of 981 08)@)(1+%) the cylinder. = 42.74 kPa (®) Upwvard motion with a negative acceleration (use “+” with =-8 m/ p= (9.81 x0.8)(3) (1+ 33) p= 434 KPa = 4rrad/sec CHAPTER FOUR 218 FLUID MECHANIK /LUID MECHANICS, CHAPTERFOUR > 49 Relative Equilibrium of Liquids TeionA ‘& HYDRAULICS Relative Equilibrium of Liquids Vain Vai is) Yan(0.6)2(1.63) -(0.6)40.7) 0.43 mx? x 1000 lit/m? 130 Liters Vertes Vets Vegas Vey ~ 400/27" 20.81) = 0497 m or 497 mm D Problem 4-11 An open cylindrical tank 1.6 m in diameter and 2 m high is full of w When rotated about its vertical axis at 30 rpm, what would be the slope of water surface at the rim of the tank? Problem 4-13 {An open cylindrical tank. 2 m in diameter and 4 m high contains water to a depth ‘of m, Itis rotated about its own vertical axis with a constant angular speed @ (@) 1f@=3rad/sec, is there any liquid spilled? (&) What maximum value of @ (in rpm) can be imposed without spilling, any liquid? (0) If = 8 rad/s, how much water is spilled out and to what depth will Solution Slope = tan 0 Slope = 2° the water stand when brought to rest? 8 (@ Whatangular speed @ (in rpm) will just zero the depth of water at the rev | 2arad | Imin center of the tank? CTaLRIne (ehe: TORS (©) If =100 rpm, how much area at the bottom of the tank is uncovered? jax trad oc = : i ()*(08) Slope = = 0.805 ryt ope = OS) - 9405 " (0) 0-3 rad/sec } Problem 4-12 (CE Board November 1978) 20. - ve ; An open cylindrical vessel 1.2 m in diameter and 2.1 m high is 2/3 full nae amy water. Compute the amount of water in liters that will be spilled out is eee i 0 am i Solution <= 90 rpm x 2/30 an rad/s (0.6) 2(981) 63m 0.815 > 07 m (some liquid spilled) 2.81) w= 6.26 rad/secx 39.78 rpm CHAPTER FOUR a 220 FLUID MECHANI carrer roun Relative Equilibrium of Liquids __& HYDRAULI ae Relative Equilibrium of Liquids (0) ©=8 rad/soe ey 208) ™ y7 158m h/2=163m>1m i By the squared property some liquid spilled but the vortex of the | of parabola paraboloid is inside the tank since < 4m. G 25 at = ey (1.58) 283 ‘Area, A = 1(0.283) Area, A = 0.889 m= blem 4 14 (CE November 1993) Another olution Nh open vertical cylindrical vessel, 2m in cliameter and 4 m high is filled with When the tank is brought to rest, the i ima fer to the top. If rotated on its own vertical axis in order to discharge @ ole el leet 2h Ne lant of water to uncover a circular area atthe bottom of the vessel 1m in Boa n & met emit re angular. ing and (b) how much water is roves 4 liameter: (a) Determine the angular speed in rpm, and (b) inthe eylindey after rotation? 0.63 m Von = cecal lution Yop = 1 (1)(0.63) Vee = 1.979" = (a) The vortex touches the bottom when =n Solve for (by squared property) or(ie 2081) a? _ 05y a as RA o= 885% 2 h-4= 025 = 646rpm o7si=4 (e) When «= 100 rpm Pee (© = 100(x/30) = 3.33x rad/sec il} 2? (3.33a)2(1)? a 2g” 20810) n= 558>4m the vortex of the paraboloid is already befow the tank (imaginary) 222 sHArTER FOUR FLUID MECHANI Relative Equilibrium of Liquids ‘& HYDRAULE ID MECHANICS CHAPTERFOUR 993 HYDRAULICS Relative Equilibrium of Liquids () Vi = Vosinds = Viet View = m (1)2(4)- Vion = 4.716 mé Problem 4-15 (CE Board November 1993) _ ‘A 1.90 m diameter closed cylinder, 2.75 m high is completely filled with having sp. gr. of 0.8 under a pressure of 5 kg/cm? atthe top, (a) What an speed can be imposed on the cylinder so that the maximum pressure at bottom of the tank is 14 kg/cm?? (b) Compute the pressure force exerted by con the side of the tank in kg, Solution h | 2750 Tim i (= 08) BHO) = 599m Unit weight of oil, y= 1000(08) Unit weight of oll y= 800 kg/m? = 0,0008 kg/cm? 6250 cm Ihe = 17500 cm = 175 m N= I~ 275- pale 175-275-625 109.75 0 (0.95) Ti = hg-2.75/2 = 173.625 m F = 800(173.625)[2n(0.95)(2.75)] F=228 x 106 kg roblam 4-16 (CE May 1985) {An open cylindrical tank having a radius of 300 mm and a height of 1.2 m is full of water. How fast should it be rotated about ils own vertical axis so that 75% of its vo)ime will be spilled out? Since 75% of the total volume is Spilled out, the paraboloid will be formed a part outside the vessel (ie, with its vortex below the tank) Voptes = Vow = 0.75f2%1.2)] Von = 09x? Vie puntole Vom pale =Yeuth-Ymaty hey > Eg. (I) cHarrer Four 224 Relative Equilibrium of Liquids HYDRA By squared property of parabola; ~ ee a >F4.@) ln Eq.) 182-2 —yiy) > nmultiply both sides by h/e Lah ey buty= 1.2 =(h-1.2)2 We (2-2.4h + 1.44) =144 © = 2287 rad/soc x © = 2184 rpm JID MECHANICS CHAPTERFOUR >> 5 HYDRAULICS Relative Equilibrium of Liquids = 12528 rad/secx = 12528 rad sec x OE x 0 = 119.64 rpm. blem 4 18 ‘open vessel, 500 mm in diameter and filled with water, is rotated about is ical axis al such velocity tal le water sun face 100 rn frots he axis takes ‘angle of 40° with the horizontal. Compute the speed of rotation in rpm. tion ‘The slope of the paraboloid at any distance “x” from the axis is given by: tano= 2x g Where 0= 40° and x= 0.1m 2 fan 40° = S— (0.1 tan 40" = 5-01) © = 907 rad/sec x 2 = 86.64xpm Problem 4-17 ‘An open cylincirical tank 1 m in diameter and 3 m high is full of water. what speed (in rpm) must it be rotated to discharge 1/3 of its content. Solution Let y be the height of the paraboloid, Since the volume of the paraboloid represents the volume of water spilled, then: Volume of paraboloid =% Full volume of cylinder an (O5Py= 4xx (05) 3) lem 439 ‘open cylindrical tank 1.2 m in diameter and 1.8 m deep is filled with water rotated aboutits own axis at 60 revolutions per minute. How much liquid spilled and what isthe pressure at the center of its bottom? 0 = 60x (n/30) = 2x xad/sec 1=12/2-06m fre GOO" 29.81), Vey = Vota Vegas = 4827 Vey = 4 (0.6) (0724) Vey = 0.409 me? =0724m, CHAPTER FOUR 226 Relative Equilibrium of Liquids Pressure at the center y y= 18-h = 18-0724 = 1.076 m Problem 4-20 A closed cylindrical vessel, 2 min diameter and 4m high is filed with wa to a depth of 3m and rotated about its own vertical axis at a constant anj speed, «The air inside the vessel is urider a pressure of 120 kPa 2 rad /sec, what is the pressure at the center and circuméore the bottom of the tank? (6) What angular speed « will just zero the depth of water at the center? 10 rad see, how much area at the bottom is uncovered? @ Ito © te Solution 81(1.076) = 10.555 kPa FLUID MECH) & HYDRAI p= Hove alee ia Af Where 0 = 40° and. ID MECHANICS CHAPTERFOUR >>5 HYDRAULICS Relative Equilibrium of Liquids rev, 60sec amd Tmin 12.528 rad /sec x 119.64 rpm lem 4-18 open vessel, 500 mm in diameter and filled with water, is rotated about is ical axis al such velonity that the water surface 100 stun frons Une axis aes angle of 40° with the horizontal. Compute the speed of rotation in rpm. lution ‘The slope of the paraboloid at any distance “x” from the axis is given by: tang= 2x Z 01m a? tan dor = 2— 0, tan 4or= 2-01) 07 rad/see x = 86.64 rpm blem 4-19 © in open cylindrical tank 1.2 m in diameter and 1.8 m deep is filled with water d rotated about its own axis at 60 revolutions per minute. How much liquid spilled and what is the pressure at the center of its bottom? Solution Figure (8) (a) Refer to Figure (b) w= rad/s 2g Faure (0) h 2g © = 60 x (n/30) = 2x rad/see .2/2= 06m = 0.724 m 20981) Vp = Vout Vepaea = Yi 92 It Vapi = ¥3 (0.6)? (0.724 ) Vapi = 0.409 mi? CHAPTER FOUR FLUID MECHA 226 UID MECHANICS CHAPTERFOUR 997 Relative Equilibrium of Liquids ‘& HYDRAUI HYDRAULICS Relative Equilibrium of Liquids Pressure at the center paw y=1.8-h =1.8-0.724 = 1.076m p= 981(1.076) = 10.555 kPa N/2=3.67m>1m (part of the paraboloid is above the vessel) Verify the positon ofthe vortex (See Page 207) Baas 2D 20) Problem 4- 8m>7.31m_«.the vortex is inside the vessel A closed cylindrical vessel, 2 m in diameter and 4 m high is filled with w toa depth of 3m and rotated about its own vertical axis at a constant ang speed, «0. The air inside the vessel is under a pressure of 120 kPa (a) Ifo= 12 rad/sec, what is the pressure at the center and the bottom of the tank? (®) What angular speed « will just zero the depth of water at the center? (©) IF@=20 rad/sec, how much area at the bottom is uncovered? ircumfere Solution y= 2h=2734) 3.83m<4m Pressure at the center, (at ©) Piz + Pai Myea-y Jy" 4-383=0.17 m 9,81(0.17) + 120 121.66 kPa (pressure at the center) }___z. ——— ora am 5 ps m Pressure at circumference, (at ©) pa yhe* Pa I= hy +h I= 017 +734=751m Figure (@) Fgure (b) (a), Refer to Figure (0; pe 981751) +120 ae ps 193,67 kPa (pressure at the circumference) 2881) CHAPTER FOUR 228 HAPTER FouR FLUID MECHANI 229 Relotve Egioum of gids HORA Relative Equiorum of les o > @=20rad/sec Let us first derive the general value of h a or? sven the vere the paaboe i Ge reches the bom a he vessel ie pe GORY cate 20.81) seast= x8 aq h=20dm BHa2eD > £4 cc) my In Figure (d): By squared propery of parabola Vera Vn mee i. ad) Yan ya ae el; ipentycaen 38a 6) wely > Bg, (b-2) By squared property of parabola: 4 | ne ae : retm eG Sobatite 0 89 (bn He f i gure (o) oat > Eq, (¢2) (FH)H=2700 A > Ea C9) Sin Substitute x#2and Xs? to Eq. (1) 2 2 2rt= [ w(y)- 7 ve (v)_-muliply both side by h/? he yP- ye Batyi=4+ The (4+ yyy 16+ 8yn- yy Inq, (c3) 281) ze 0 = 12528 rad/sec x 2 welp = 1196 rpm oe xe = pq GD = 0:52 Area nx = x(0.152) Area=0.48 m? (area uncovered at the bottom) 230 Problem 4-21 A closed vertical cylindrical vessel, 1.5 m in diameter and 3.6m high is full of brine (5 = 1.3) and is revolved about its verticalaxis with a cons angular speed. The vessel is made up of steel 9 mm thick with an alloy tensile stress of 85 MPa and has a small opening at the center of the top € (@) If the angular speed is 210 rpm, what is maximum the stress in the w CHAPTER FOUR Relative Equilibrium of Liquids ‘CHAPTER FOUR Relative Equilibrium of Liquids 231 Since l= 13.86 > 7.2, the vortex is below the vessel, See Figure (6) (0) To what rata angular seed cant vest be revolved? a= Vareest E09) = Yount yn- Yer? yn ‘Solution 18 Py - xr ye > Eq. (1) (a) @=210rpmv 9/30 Purad/s By squared propery of parable pooh " is =. i i > Fg.) T ; T ' 1 Substitute 232 and 37 to Bq, (1) ‘ y 182 © wiy)- "2 y(n) > mullply bot sides by 1/2 | : ‘ fi 1 Rb yea ne i ape Buty) =3.6+ ys ee ae ee 18h = 8.6 + yo) -y2 ae | 18h=12.96 +7.2)n+ y2-92 | Taya tah 13% 2e.®) 7.2yp = 1.8(13.86) - 12.96 ‘ pe1eesin owe Pe) | eas f= 12195 m Shear p= 9.si013\121%5) pa 15550 KPa = £155.52)(1500) _ (@n)?(0.75)? pe) Te oat) ‘Sp 12,960 kPa S,= 12.96 MPa (maximum wall stress) chaerer Four FLUID MECHA ‘CHAPTER FOUR 232 _Relave Equilbrium of Liquids &HYDRAI Relative Equilibrium of Liquids 233 (b) For maximum value of a, 5, = 85 MPa p2(1.8x10°) Rae | 82x 10'= 85x 10'= oeesrs) 26) 555 KPa y= 79.98 m e n= h-79.98 pre yin 55.5 = 9.81(1.6)hz In Eq. (4) Tyg = 29.02 m 72(h- 79.98) = 1.8 - 12.96 is r/2-27 3 ~ 562.896; r= 104.24 m i= 2902-27 -1561 pam Ji-1071m 2; (0.9) i - Bah rae (075)? vo424 29.81) = 16.1 rad/sec x 2 © = 603 rad/sec » 30/x = 153.8 rpm (maximum allowable angular speed) ©=576 rpm pea blem 4-23 Problem 4 1.5 m diameter impeller of a closed centrifugal water pump is rotated at AL. m diameter closed cylinder, 27 m high is completely filed with gly rpm. If the casing is full of water, what pressure is developed by having sp. gr. of 1.6 under a pressure of 245 kPa at the top. The steel pla tion? which form the cylinder are 5 mm thick with an ultimate tensile stress of MPa, How fast can it be rotated about its vertical axis to the point of bursting tion Solution \ Pressure head, ~~ for h le @ = 1500 x 2/30 iow aoe © = 50n rad/sec y 9.81(1.6) a o FB -1561m 2(9.81) 2. 2707.4 m of water 581 The maximum tensile stress occurs ee at point @ pD From Been oad iz ru mec /LUID MECHANICS. CHAPTERFOUR 736 Relative Equilibrium of Liquids’ ‘& HYDRAUI ‘6 HYDRAULICS Relative Equilibrium of Liquids Problem 4 - 24 (CE Board) olution A conical vessel with sides inclined AO® with its vertical axis is revolved a another axis 1m from its own afd parallel. How many revolutions: minute must it make in order that water poured into it will be entit discharged by the rotative effect? payin = Solving for ls In=ye Solution aol 2g The water in the vessel will entirely be discharged at a speed when the paraboloid is tangent to the cone at the vertex, hence, the inclination, 0, of the paraboloid at x= 1 mis 60° orits slope is | [257-57] tan 60° i I= 231.27 m. p= (981 x 0.829)(231.27) Pointof From the formula: tan {A glass U-tube whose vertical stems are 300 mm apart is filled with mercury Jo a depth of 150 mm in the vertical stems. It is rotated about a vertical axis ough the midpoint of the horizontal section. What angular speed « will ‘produce a pressure of absolute zero in the mercury at the axis? tan 60° = = 4:12 rad/sec © 39.36 revolutions per minute Solution . 7 : Problem 4-25 (CE November 1992) Bane Dee ee tte an et co ae then capped, and placed on a horizontal position. It is rotated at 2755 radi about a vertical axis 0.5 m from one end (outside the pipe). What is pressure in kPa at the far end of the pipe? Since the pressure at the center is absolute zero, then the gage pressure at the center is -Paum OF y= 076+ 0:5 191m 070.15)" “Z981) © =2817 rad/sec x 2 269 rpm ogi = 236 P40) By squared propery of parabola: EOE wo Ya n — (04? (02)1= 02 Pre Ta naan > 4.2) = 572ra/secx Substitute y to Bq, (1) o= 546:pm rane 1s y= 025 ox? Problem 4 - 28 n= 2 "glass tubing consist of5 vertical stems which ae 500m apart connected fie a single horizental tube. The tube is filled with water to a depth of 500 many ante the vertical stems. How fast should it be rotated about and axis through, 20.81) middle stem to just zero the depth of water in that stem? @ = 443 rad/sec x @=423 rpm ‘CHAPTER FOUR On 238 “LUID MECI ID MECHANICS CHAPTERFOUR 230 Relative Equilibrium of Liquids & HYDI HYDRAULICS. Relative Equilibrium of Liquids Problem 4 - 29 'A75 mm diameter pipe, 2 m long is filled with water and capped at both. It is held on a plane inclined 60° with the horizontal and rotated ak vertical axis through its lower end with a constant angular speed of 5 rad) (@) Compute the pressure at the upper end of the pipe and (t) determing ‘minimum pressure and its location in the pipe Minimize i, differentiate Eq, (I) with respect to x and equate to zero: a 548x ~ tan 60° = 0 de x= 068m a= xsec60" = 0.68 sec60? = 1.36 m Solution Siocs thar is nota presere | ihe Pe Wap Hed a delovedelol Goppe dt! seals etn Hee preorteee GAN Sa feaeeee rested 7 In Eq. (1): I= 1.274(0,68)! + 1.73 (0.68) tanou" W=114im min 981(1141) nin = 11.196 kPa located 1.36 m from the lower end (along the pipe) lem 4-30 paraboloid will be 1.73 m above the lower end : cylindrical bucket 150 mm in diameter and 200 mm high contains 150 mm of ot? ater. A boy swings the bucket on a vertical plane so that the bottom of the Whe 1 Icket describes a circle of radius 1m. How fast should it be rotated so that cats ‘water will be spilled? = FO) 2(9.81) lution he1274m (a) Pressure at the upper end! i 81(1.274) 2.497 kPa (®) Minimum pressure Pom yh Solve for ht Way+2 = 173-x tan60” ihe 42 Figure () i 1m 2742 W'=1.274 32+ (1.73-xtan60") > Eg. (1) ee Figure (@) Sate ron Pate Pticrar tvs 240, Mme cin ot ii es par Miexcucosr, 241 previous chapters deals only with fluids at rest in which the only yrificant property used is the weight of the fluid. This chapter will deal with lds in motion which is based on the following principles: (a) the principle of jnservation of mass, (b) the energy principle (the kinetic and potential ges), and (c) the principle of momentum. 0? (0.925) = 981 26 rad/sec « & 31.13 xpm Problem 4- 31 AA cubical tank is filled with 2 m of oil having sp. gr. of 0.8. Find the fo acting on one side of the tank when the acceleration is 5 m/s? (a) verti upward, and (b) vertically downward ISCHARGE OR FLOW RATE, Q hharge or flow rate is the amount of fluid passing through a section per tof time. This is expressed as a mass flow rate (ex. kg/sec), weight flow rate KN/sec), and volume flow rate or flow rate (ex. m*/s, lit/). Solution Volume flow rate, Q= Av ‘Mass flow rate, M=p Q Weight flow rate, W= yQ (a) P= pa Fly he tw) A 981 x08)(1) +5/981)(@) 9) 7.392 KN where Q= discharge in m/s or ft/s ‘A= cross-sectional area of flow in mor ft = mean velocity of flow in m/s of ft/s p= mass density in kg/m’ or slugs/f y= weight density in N/m? or lb/ft (Pape Fe byl af) A (981 » .8)(1)1 -5/9.81)(2%2) 5.392 kN |FINITION OF TERMS uid Flow may be steady or unsteady; uniform or non-uniform: continuo dinar or turbulent; one-dimensional, two-dimensional or three-dimensional; and Wlational or irotational CHAPTER FIVE 242 FLUID MECI JID MECHANICS CHAPTERFIVE Day Fundamentals of Fluid Flow ‘&HYDI HYDRAULICS Fundamentals of Fluid Flow. st ‘This occurs when the discharge Q passing a given cross-section, time. Ifthe flow Q at the cross-section varies with time, the flow is unsteady dy Flow flow is said to be turbulent when the path of individual particles are gular and continuously cross each other. Turbulent flow normally occurs yen the Reynolds number exceed 2,100, (although the most common ation is when it exceeds 4000). Uniform Flow ‘This occurs if, with steady flow for a given length, or teach, of a str average velocity of flow is the same at every cross-section. This ustally ‘when an incompressible fluid flows through a stream with uniform section. In stream where the cross-sections and velocity changes, the fl said to be non-uniform. yminar flow in circular pipes can be maintained up to values of Ras high as 1000, However, in such cases this type of flow is inherently unstable, and least disturbance will transform it instantly into turbulent flow. On the er hand, itis practically impossible for turbulent flow in a straight pipe to sist at values of R: much below 2100, because any turbulence that is set up ll be damped out by viscous friction. Continuous Flow ‘This occurs when at any time, the discharge Q at every section of the stre re ew the same (principle of conservation of mass). is occurs when in an incompressible fluid, the direction and magnitude of velocity at all points are identical -Dimensional Flow ls occurs when the fluid particles move in planes or parallel planes and the samline patterns are identical in each plane. Figure 5-1 ‘ Continuity Equation For incompressible fluids: yese are imaginary curves drawn through a fluid to indicate the direction of jotion in various sections of the flow of the fluid system, v3 = constant Streamtubes These represents elementary portions of a flowing fluid bounded by a group ‘f streamlines which confine the flow For compressible fluids: pido = pada = polity = constant Fa. 5 cor_wAvor = yaAce2 = jpAss = constant FFlow Nets These are drawn to indicate flow pattors in case of two-dimensional flow, or Laminar Flow pare a ‘The flow is said to be laminar when the path of individual fluid particl not cross or intersect. The flow is always laminar when the Reynolds nui is less than (approximately) 2,100, 244 SHAPTERFIVE—_ FLUID MECHAR Fundamentals of Flutd Flow & HYDE ENERGY AND HEAD The energy possessed by a flowing fluid consists of the kinetic and the pot energy. Potential energy may in turn be subdivided into energy de position or eleontion above a given datum, and energy due to pressure i fluid. The amount of energy per pound or Newton of fluid is called the hes Kinetic Energy The ability of the fluid mass to do work by virtue ot its velocity B-iMenn ee - 5 Kee or vlasty cts EE 3 al We 3g For circular pipe of diameter D flowing full: (gsaP _ _o? 2g 2gA? 2g got Elevation Energy (Potential Energy) ‘The energy possessed by the fluid by virtue of its position or elevation respect toa datum plane. Blevation Energy = We = Mgz 5-10 Hlévation ead = Rene tnery a2) gb IO MECHANICS pHYORAULICS CHAPTER FIVE Fundamentals of Fluid Flow 245 jure Enoray (Potential Energy) sider a closed tank filled with a fluid which has a small opening atthe top fhout pressure at the top, the fluid practically will not flow. In Chapter 2 tquivalent head (pressure head) for « pressure of p is p/+. Hence the gure energy is equivalent to: Prossure Bnergy = W Eq. 5-12 Pressure Energy Ww Pressure head zg Eq.5-13 7 where position of the fluid above (+) or below (-) the datum plane. p> fluid pressure = mean velocity of flow | Flow Energy, E we total energy or head in a fluid flow is the sum of the kinetic and the tential energies. It can be summarized as: Tplal Energy = Kinetic Energy + Potential Energies Eq.5-14 Da 5.915 1 Total Head, }OWER AND EFFICIENCY Power is the rate at which work is done. For a fluid of unit weight y (N/m!) And moving at a rate of Q (@?/s) with a total energy of E (m), the power in N In/s (oule/sec) or watts is Power=Q7E Eq.5-16 Efficiency, = ee 100% Bq.5-17 Note: 1 Horsepower (hp) = 746 Watts 1 Horsepower (ap) = 550 ft-b/sec 1 Watt = 1 N-m/s= 1 Joule/see CHAPTER FIVE FLUID MECI 246 Fundamentals of Ftd Flow HYDRA UID MECHANICS, CHAPTER FIVE p47 HYDRAULICS. Fundamentals of Fluid Flow nergy Equation with Head Lost: Jonsidering head lost, the values that we ean attain are called actual onfues ith reference to Figure 5-4 BERNOULLI'S ENERGY THEOREM The Bernoulli's energy theorem results from the application of the prey onservation of energy. This equation may be summarized as follows: Bernel’ Pipe, mn physics, he can > hates the speed of 2 moving Ml 35) meeases, ta pressure wit that Aecesses, Orginal forded in 1738 Sissmathematlan ang piss Oa Boreal states ta he ol ong $eeay fing ld asters sonata slong the fow path, A nerease he ‘ond mist therefore be matched by a Ey-Hla= Es Envationt + Enidet “Enatcroirned= Ewan Eq. 5 Eneray Equation without Head Lost: If the fluid experiences no head lost in moving from section 1 to section 2 the total energy at section 1 must be equal to the total energy at sect Neglecting head lost in fluid flow, the values that we get are called theoretical values. With reference to Figure 5-3: inorgy Equation with Pump: Pump is used basically to increase the head. (Usually to raise water from a Hower to a higher elevation), The input power (Papu) of the pump is electrical Iehergy and its output power (Powys) isthe flow energy. Figure 5-3 24g CHAPTER FIVE FLUID MECH) MEcHANtes charrer Five Fandamental of Rut Fow SORA 249 DRAULICS Fundamentals of Fluid Flow Ty +HA-HLa= Ee acteristics of HGL + HGL slopes downward in the direction of flow but it may rise or fall due to changes in velocity or pressure. + For uniform pipe cross-section, HGL. is parallel to the EGL. For horizontal pipes with uniform diameter, the drop in pressure heads also equal to the head lost between these v2 ae : a atHls 84.5 2g 7 + Pat HA R Output Power of Pump = Qy HA Energy Equation with Turbine or Motor: Turbines or motors extract flow energy to do mechanical work which in converted into electrical energy for turbines ° rgy Grade Line (EGL) gy grade line is a graphical representation of the total energy of flow (the of kinetic and potential energies). Its distance from the datum plane is + eee acteristics of EL. + EGL always slope downward in the direction of flow, and it will only rise with the presence of a pump. ‘+ The drop of the EGL between any two points is the head lost between. those points. + For uniform pipe cross-section, EGL is parallel to the HGL. TUREINE Figure 5-6 + EGL is always above the HGI. by an amount equal tothe velocity head, B-HE-Hba= Es ye es 2 » esta een ees + Neglecting head loss, EGL is horizontal tae Be htt Hla tHE 4 lecting , 2g 7 2 na _Input Power of Turbine = Qy HE ENERGY AND HYDRAULIC GRADE LINES Hydraulic Grade Line (HGL) Also known as pressure gradient, Itydnaite grade line 13 the graphi representation of the total potential energy of flow. It is the line that conne the water levels in successive piezometer tubes placed at intervals along, pipe. Its distance fom the datum plone is Bs i Figure 5 - 7: Ilustration showing the behavior of energy and hydraulic grade lines, teres \ Bea Saves Riccrie 250 gear Nanay ie Eromastie Pcs 251 Solved Problems Problem 5-4 Water flows through a 75 mm diameter pipe at a velocity of 3 m/sec. Hi the volume flow rate in m/sec and lit/see, (6) the mass flow rate in kg and (c) the weight flow rate in N/sec 1" 79.3(80+273) Solution 20 = 12.39(0.16(032)]0 1.53 mys (average velocity) @ Q=Av ae 4 (0.07570) (0:16)(0.32)1.53) = 0.013 mY/s x 1000 lit/m? Q=1614 msec (volume flux or discharge) Q= 13 liysee ® |M=eQ jem 5-4 ae 100-mm diameter plunger is being pushed at 60 mm/sec into a tank filled M=13 kg/see (mass flow rate) ith oil having sp. gr. of 0.82. If the fluid is incompressible, how many N/s of lis being forced out at a 30-mm diameter hole? o ition W =127 N/sec (weight flow rate) ce the fluid is incompressible: -Q: a 2 Problem 5-2 (CE November 1995) as Whats the ae of ow of water pesing hough a pipe witha diameter fatale tvs Vs eee | (0:1)x0.06) 00047 m*/s v= 008 mys a Solution (Qs = 0.00047 m?/s . Flow rate, Q= Av w-7Q ree J mmo Q= $(0.02}205) (6810 x 0.82)(0.00047) = 157 x 104 mysee i" STEVE mss > a et Ifthe velocity of flow in a 75.mm diameter fire hose is 0.5 m/s, what is the Velocity in a 25 mm diameter jet issuing from a nozzle attached at the end of the pipe. Compute also the power available in the jet ‘Problem 5-3 Air at 30°C and 110 kPa flows at 20 N/s through a rectangular duct measure 160 mm x 320 mm. Compute the average velocity and volume Use gas constant R= 29.3 m/*K. ‘CHAPTER FIVE 252 FLUID MECI CHAI CHAPTER FIVE NICS Fundamentals of Fluid Flow oe 253 IRAULICS Fundamentals of Fluid Flow Solution g By continuity equation PE=Wa nose = Qu W=7x Volume 981 $6710) W=1,926.2kN PE= 192627 PE = 13,483.32 kN-m Aun ¥ (0.075) (05) = $ (0.025) 0, Y= 45mys (velocity of the et) Power, P= QyE Q=Av Q~ § (0.025) (45) = 0.002209 m?/s pe 2 4F Lica m harging through a 50-mm diameter nozzle. ig” 21981) lution Kinetic energy flux = Kinetic Energy per second = Power Power, P = 0.002209(9,810)(1.0821) Power, P= 22.37 watts (power available in the jet) Power, P= Oy E Problem 5-6 iment A turbine is rated at 600 hp when the flow of water through itis 0.61 Assuming an efficiency of 87%, whats the head acting on the turbine? Q__ 002 Solution oe F057 Given: Power output ae ‘v= 10.186 m/s Efficiency, n 2 p= 10186)" = 5258 m 2(981) Power inpu 02(9810 x 0.85)(5.288) Power inp Power input = Qy HE 514,488 = 0.61(9,810)HE joglecting air resistance, determine to what height a vertical jet of water could le if projected with a velocity of 20 m/s? Problem 5-7 A standpipe 5 m in diameter and 10 m high is filled with water. Calculate potential energy of the water if the elevation datum io taken 2 m below base of the standpipe 254 SHATERFIVE \ FLUID MECH) CHAPTER FIVE Fundamentals of Fluid Flow 255 {YDRAULICS: Fundamentals of Fluid Flow Solution AAs the jet rises, its Kinetic energy is transformed into potential energy pipe carrying oil of specific gravity 0.877 changes in size from 150 mm at jon 1 and 450 mm at section 2, Section 1 is 3.6 m below section 2 and the sures are 90 kPa and 60 kPa respectively. If the discharge is 150 lit/sec, sine the head lost and the direction of flow. Neglecting air resistance ition Q:=015 m/s = 0943 m/s #045)" ‘Taking © as datum: Problem 5-10 peo Bd Peg 90 Water is flowing in an open channel at a depth of 2 m and a velocity of 3 2g 7 20.81) 81x07 It flows down a chute into another channel where the depth is 1 ma E,= 14.135 m velocity is 10.m/s. Neglecting friction, determine the difference in elevate the channel floors pete en Solution 2g ¥, emt 2s eee 20981) * Gax087) E,=10.62m Since F; > Fy the flow is from 1 to 2 Head Lost, HL = Ey - Ey = 14.135 - 10.62 Head Lost, HL = 3515 m Neglecting friction (head lost): ‘ete ay 28) 20.81) 3.64 m cuapren rive eres hii id 256 Problem 5 - 12 Oil flows from a tank through 150 m of 150 mm diameter pipe and then discharges into air as shown in the Figure. If the head loss from point 1 to point 2 is 600 mm, determine the pressure needed at point 1 to cause 17 lit/sec of oil to flow UID MECHANICS. CHAPTER FIVE 57 HYDRAULICS. Fundamentals of Fluid Flow M= pA pal(0:3)(0.3)]@) = 0.1875 p2= 0.875 kg/m’ (mass density at section 2) blem 5-14 ater flows at the rate of 7.5 m/s through 75-mm diameter pipe (pipe 1) and wos through 50-mm diameter and 65-mm diameter pipes at the vate of 3 V/s and 35 m/s, respectively as shown in the Figure.. Air at the top of the unk escapes through a 50-mm-diameter vent. Calculate dii/dt and the Solution Q= 0017 m/s Energy equation between @ and @: Ey Hla = E> eA a. _ 80.017)? x? g(0.15)* +0430 = 10.65 m of oil P= 10.65(9.81 » 0.84) ~ 87.76 kPa ‘Problem 5-13 — Gas flowing trough a square conduit whose section gradual ss section gradualy changes 180 mm ection 1 t0 300 mm (ction 2) At ston the wee of ane in/s an the density of gas s/n whe at section 2 he velo of owe an/5. Calculate the ns ow ate and the density ofthe gar aston 2 Solution ‘Assuming the flow to be incompressible: Qn = Qoat 4} (0.075)%7.5) = (0.08)%3) + § (0.065)2.8) + 5 (0.6)Pan/at ddi/dt = 0.0553 m/s Solution ‘ a) 300mm Considering the air above the tank: i : i: 1Q1 Quel a a le 5 (005)204= 5 (0.6) dh/at 5 (0.05) 04 = 4 (0.6)°(0.0553) %¢= 7.963 mys. (velocity of air flow) = 110.15)(0.19)17) 1575 kg/sec (mass flow rate) cuarter rive | FLUID Mec CHAPTER FIVE 258 Fundamentals of Hid Flow &HYDRAU Fundamentals of Fluid Flow 299 Problem 5 - 15 A liquid having sp. gr. of 20 is flowing in a 50 mm diameter pipe. The head at a given point was found to be 17.5 Joule per Newton, The elevati the pipe above the datum is 3 m and the pressure in the pipe is 65.6 ‘Compute the velocity of flow and the horsepower in the stream at that poi (0.03)? Solution 2 57 (9.81)(0.2)" Total energy, B~ 5 + Boz 8(0.08)" ane 2g 77(9.81)(0.15)* 175 Joule/N x (1 N-m/Joule) Energy Equation between A and B: M8" 3g * B10) *° Pang BA + ay Hils + HA Hilos™ 1.156 0 0410-20065) + HA 100287) = 0+ 0+ 60 28 HA=51.563 m 2= 1479 mys _ (velocity of flow) Power output = Q 7 HA = 0.03(9,810)(51.563) = 15,175 watts « (1 hp/746 watts) Power output = 20.44 horsepower (rated power of the pump) Power, P= QyE [§ (0.05)14.79)] « (9810 x 2) «175 9970.92 watts x (1 hp/746 watts) Power, P= 13.37 hp ' Energy Equation between A and 1: Eq-Hlaa= Ey 2 ve Ban 4 PA + 24 Higa = ot Pe; 2g Pressure heads at 1 and 2: Problem 5 ~ 16 (CE May 1994, May 2004) ‘The pump shown draws water from reservoir A at elevation 10 m and lifts reservoir B at elevation 60 m. The loss of head from A to 1 is two times 0+0+10 -2(0.0465) velocity head in the 200 mm diameter pipe and the loss of head from 2 to 10465 + ten times the velocity headin the 150 mm diameter pipe. Determine the Fh ase ot alee horsepower ofthe pump and the pressure heads at {and 2in meters when Y discharge is 0.03 mt /see Energy Equation between of 2 and 8 non pe oa 7 42 Hines Ee Ea 22 +0. 10(0187)=0+0+60 Y = 61,323 m of water CHAPTERFIVE 6 4 CHAPTER FIVE FLUID MECHA 260 _ Fundamentals of Fiuid Flow ‘© HYDRAU Fundamentals of Fluid Flow Problem 5 - 17 (CE November 1986) j Hia= 916013 @ A pipeline with a pump leads to a nozzle as shown. Find the flow rate (2a pump develops an 80 ft (24.4 m) head. Assume head lost in the 6-inch fs = = 24508 0? mm) pipe to be five times its velocity head while the head lost in the 2g” x g(00763) (102 mm) pipe to be twelve time its velocity head. (2) Compute the flow tok + 2A 9160730! = 1508 G+ 0+ 244 i orl aunerntne eae Crt feta oar ats Tae ote pressure head atthe suction side. Q=00ns mii, > Dackaree awasam ee (b) Energy and Hydraulic grade lines aire v2 _ 80.0415)? 2g xi(oi)(0.152)" aren Line? v2 goons)? 2g 7 (9.81)(0.102)* ust, 8(0,0815)" 90°(15.2 m) 2g -n°(9.81)(0.0762)* HL, = 773.96 Q? = 1.33 m Solution Hla= 9,160.13 Q? = 15:78 (a) Discharge bt Q=O:=00=0 “toe Energy Equation between A and B: oF y= 15.780 E_- HL + HA- Hla = Ep “Y va? aay ita = 331 fA 4 PA 4 2, - HL) +HA-Hla= 2B + Be Wry: 2g HA= 244m se G13m) ¥7(981)(0.152)" HL = 773.96 ee Ot + (9.81)(0.102)* CHAPTER FIVE 262 Fundamentals of Fluid Flow (6) Pressure head at $ Energy Equation between A and $ E4-Hly=Es PAs PA yay = OS 2s 1 2g og oy 2s a 2 = 0266 m 2g” 2g 0+0+213-133=02664 #5 +152 Y BS = 4504m 7 (Or from the figure shown above, the pressure head at Sis the vertical dist from the pipe to the HGL. Ps =19.704-152 Y PS 24504 m Y Problem 5-18 (CE November 1980) Water enters a motor through a 600-mm-diameter pipe under a pressure kPa. It leaves through a 900-mm-

You might also like